Как решить квадратное: Квадратные уравнения — урок. Алгебра, 8 класс.

Содержание

Квадратные уравнения 8 класс онлайн-подготовка на Ростелеком Лицей

Составление уравнений

 

Когда вы просыпаетесь утром и слышите за окном размеренный стук капель, то сразу понимаете, что на улице идет дождь. Для этого вам даже не надо выглядывать в окно. Или мама приготовила вам с братом 5 бутербродов в школу. Увидев только 2 бутерброда, вы понимаете, что ваш брат взял с собой 3, хотя и не видели, как он это сделал.

 

В жизни мы часто сталкиваемся с такими ситуациями: наблюдаем одно, а на основании этих наблюдений делаем выводы о другом. Если речь идет о числовых величинах, то по результатам наблюдений мы можем составить уравнение для получения вывода – нахождения неизвестной величины.

Как измерить толщину листа бумаги? Обычная линейка не подойдет – у нее цена деления больше измеряемой величины. Но можно воспользоваться тем, что толщина у листов, обычно, практически одинаковая. Значит, если взять много листов, то толщина одного – это толщина пачки, разделенная на количество листов в ней.

Получаем метод измерения: взять пачку такой толщины, чтобы ее можно было достаточно точно измерить имеющейся линейкой, затем посчитать количество листов в ней. Если, к примеру, толщина пачки из 500 листов оказалась равной , то получаем уравнение:

Откуда толщина одного листа:

Другой пример. Вам нужно посчитать, сколько конфет лежит в пакете. Конечно, это можно сделать напрямую. Ну, а если конфет очень много? Выход есть! Если мы знаем массу одной конфеты (например, на упаковке написано: 15 г), то можем взвесить весь пакет (пусть получилось 1800 г). Обозначив количество конфет за , составляем уравнение:

Решая уравнение, получаем ответ:

 

Квадратные уравнения

 

 

Полученные в примерах уравнения:  и  – это линейные уравнения (уравнения вида ). С ними и с задачами, которые ими описываются, мы уже умеем работать.

 

Но в линейных уравнениях переменная всегда в первой степени (. Понятно, что так будет не всегда. Например, если мы ищем сторону квадрата с площадью , то должны решить уравнение: , которое уже не будет линейным (логично так и назвать его – нелинейным).

Многие задачи могут быть смоделированы нелинейными уравнениями. Например, для нахождения минимальной начальной скорости мяча , с которой нужно его подбросить, чтобы он перелетел через забор высотой  метра, нужно решить квадратное уравнение .


 

Как получилось такое уравнение?

Воспользуемся формулой из курса физики, а именно – формулой для вычисления расстояния, которое прошло тело при равноускоренном движении.

Когда мы подбрасываем мяч, то на него действует только сила тяжести, т.е. мяч движется с ускорением , которое направлено вниз. Пока мяч летит вверх, это ускорение замедляет его начальную скорость до  (в верхней точке), а когда он начинает падать, наоборот, разгоняет (увеличивает скорость).

Рис. 1. Когда мяч начинает падать, ускорение увеличивает его скорость

В этом случае расстояние от земли до мяча можно вычислить по формуле:

где  – начальная скорость мяча,  – скорость мяча на данной высоте,  – ускорение свободного падения:

Мяч перелетит через забор, если высота его подлета станет равной высоте забора:

 м

Т. к. мы ищем минимальную скорость, то достаточно, чтобы это была верхняя точка траектории, т.е. скорость мяча в ней равнялась

Кроме того, мы обозначили:

Получаем:

Откуда:

Подробнее о решении таких задач (и о том, откуда взялась использованная нами формула) вы узнаете на уроках физики в 9 классе


Рассмотрим два линейных уравнения:

Если мы их перемножим, то получим уравнение:

Понятно, что у этого уравнения два корня:  и , потому что произведение равно  только тогда, когда хотя бы один из множителей равен .

Если мы раскроем скобки в левой части, то получим уравнение:

Мы получили пример простейшего нелинейного уравнения – квадратного уравнения.

Строгое определение: квадратное уравнение – это уравнение вида:

где  – заданные числа (коэффициенты квадратного уравнения), причем, ведь если , то уравнения будет линейным .

 

Алгоритм решения квадратных уравнений

 

 

В рассмотренном нами примере квадратное уравнение  можно решить, разложив левую часть на множители:

 

Но для любых ли  можно разложить квадратный трехчлен (так называется выражение   в левой части квадратного уравнения – три члена – три слагаемых, старшая степень – квадрат) на линейные множители?

Например,  разложить на множители нам не удастся, у уравнения  нет действительных корней (потому что, как мы знаем, квадрат действительного числа не может быть отрицательным: ).

Но можно ли как-то определить наличие или отсутствие корней квадратного уравнения по его коэффициентам? Оказывается, да. И это мы сегодня тоже научимся делать.

Итак, как решать квадратные уравнения? Один способ мы уже нашли – попытаться разложить левую часть на линейные множители, и приравнять каждый из них к . Алгоритм будет следующий:

  1. перенести все слагаемые в одну сторону;
  2. разложить полученное выражение на множители;
  3. решить полученные линейные уравнения.

Для разложения на множители, можем использовать различные уже известные нам приемы:

  1. вынесение множители за скобки;
  2. формулы сокращенного умножения;
  3. метод группировки;
  4. выделение полного квадрата.

Повторить эти методы вы можете, посмотрев урок «Разложение многочленов на множители»

Рассмотрим несколько примеров.

 

Пример 1. Решить уравнение:

Решение.

Перенесем слагаемое из правой части уравнения в левую:

Представим число так:

Тогда:

Применяем формулу разности квадратов:

Откуда:

Часто в квадратных уравнениях получается  ответа, поэтому возле неизвестной ставят индексы и записывают так:

Ответ: .

 

Пример 2. Решить уравнение:

Решение.

Выносим общий множитель за скобки:

Тогда:

Ответ: .

 

Неполные квадратные уравнения

 

 

Рассмотренные квадратные уравнения называются неполными квадратными уравнениями. Если вы сравните их с общим видом квадратного уравнения: , то поймете, почему.

 

Так, в уравнении  отсутствует слагаемое с , т.е. в нем коэффициент . В уравнении  отсутствует свободный член, т.е. . Рассмотрим еще несколько примеров неполных квадратных уравнений.

 

Пример 3. Решить уравнение:

Решение.

Чтобы использовать здесь формулу разности квадратов, вспомним соотношение для квадратных корней:

для любого неотрицательного . Соответственно:

Тогда:

Ответ: .

 

Пример 4. Решить уравнение:

Решение.

Формулы для суммы квадратов нет, поэтому мы не можем разложить левую часть уравнения на множители. В таком случае, уравнение не имеет решений. Покажем это:

Квадрат любого действительного числа всегда неотрицательная величина, значит, нельзя найти такое значение , при котором .

Ответ: нет действительных корней.

С примером ситуации, когда квадратное уравнение не имеет решений, можно ознакомиться ниже.


 

Пример задачи, которая не имеет решения

Уравнения возникают, как модели для решения некоторых задач. Понятно, что некоторые задачи могут не иметь решения, а, значит, не будет иметь решения и соответствующее уравнение.

Вернемся к примеру с мячом, который бросают вертикально вверх. Выше мы говорили о формуле для пройденного мячом расстояния:

Если воспользоваться тем, что скорость при таком движении изменяется по формуле: , то получим:

Тогда, если тело подбросили вертикально вверх со скоростью  м/с, то зависимость высоты над поверхностью  будет иметь вид:

Чтобы определить время, через которое тело будет находиться на высоте  метра, нужно будет решить уравнение:

На высоте  м – уравнение:

Но тело, брошенное вертикально со скоростью  м/с, не долетит до высоты  метров (максимальная высота составит  метров). Поэтому вполне естественно, что уравнение  не будет иметь решений.

Когда мы говорим о том, что квадратное уравнение не будет иметь корней, то всегда будем говорить о действительных (вещественных) корнях. Мы говорили, что можно расширить такой инструмент число и ввести числа, квадрат которых может быть отрицательным (см. рис. 1):

Рис. 1. Действительные и комплексные числа

Такие числа называются комплексными. Если рассматривать решение квадратного уравнения на множестве комплексных чисел, то у него всегда будет два корня. Например:

Но, если по определению:

то:

Тогда:


 

 

Решение квадратных уравнений

 

 

Рассмотрим еще несколько примеров квадратных уравнений.

 

 

Пример 5. Решить уравнение:

Решение.

Здесь видим формулу полного квадрата:

Ответ: .

 

Пример 6. Решить уравнение:

Решение.

ФСУ здесь не видно, поэтому применим метод выделение полного квадрата. Квадрат первого выражения уже есть . Далее должно идти удвоенное произведение: . Глядя на выражение, видим, что вместо знака вопроса должно быть  (чтобы получить ):

Для полного квадрата не хватает квадрата второго выражения. Добавим и вычтем его:

Не забудем о последнем слагаемом :

Получим разность квадратов :

Ответ: .

 

Дискриминант квадратного уравнения

 

 

Итак, раскладывая на множители левую часть, мы можем решить любое квадратное уравнение. Если же разложить на множители нельзя, то уравнение не будет иметь решений. Это один из методов решения. Но, обратите внимание, он эффективно работал, когда нам удавалось легко разложить левую часть на множители.

 

Универсальный метод – это метод выделения полного квадрата, но, как мы видели на примере, он может быть достаточно громоздким. Попробуем с его помощью вывести готовую формулу для вычисления корней квадратного уравнения по его коэффициентам.

Квадратное уравнение в общем виде  можно преобразовать к виду:


 

Преобразование квадратного уравнения

Квадратное уравнение:

Поскольку , можем разделить обе части уравнения на :

Второе слагаемое должно представлять из себя удвоенное произведение:

Выделим полный квадрат – прибавим и вычтем :


Рассмотрим подробнее вторую дробь:

Поэтому знак дроби определяется знаком выражения . Это выражение называют дискриминантом квадратного уравнения:

В зависимости от знака дискриминанта, получаем разные решения:

1. если , то можно записать:

Тогда:

Получаем:

Используем формулу разности квадратов:

Получаем корни:

В этом случае уравнение имеет два корня. Их можно записать одной формулой:

2. если , то:

Получаем:

Получаем один корень уравнения. Иногда еще говорят, что «уравнение имеет два совпадающих корня». Полученное выражение для корня уравнения согласуется с формулой для квадратных корней при положительном дискриминанте:

3. если, то разложить на множители левую часть не удастся. При этом квадратное уравнение не будет иметь корней. Покажем это:

В левой части стоит неотрицательное выражение. В правой части – отрицательное:

Таким образом, уравнение не имеет решений в действительных числах.

Решим несколько квадратных уравнений, используя полученные формулы. Начнем с уравнения, которое мы уже решали.

 

Пример 6*. Решить уравнение:

Решение.

Сравнивая с общим видом уравнения , выпишем коэффициенты:

Считаем дискриминант:

Дискриминант положительный, значит, уравнение имеет два корня:

Естественно, получаем те же корни, что были получены при решении другим методом.

Ответ: .

 

Пример 7. Решить уравнение:

Решение.

Сравнивая с общим видом уравнения , выпишем коэффициенты:

Считаем дискриминант:

Дискриминант , уравнение не имеет решений в действительных числах.

Ответ: нет действительных корней.

С решением еще одного квадратного уравнения вы можете ознакомиться ниже.


 

Решение еще одного квадратного уравнения

Пример. Решить уравнение:

Решение.

Выпишем коэффициенты:

Считаем дискриминант:

Дискриминант положительный, значит, уравнение имеет два корня:

Используем свойства корня, чтобы упростить выражения (см. урок «Свойства квадратного корня»):

Получаем:

Ответ: .


 

 

Теорема Виета

 

 

Чтобы рассмотреть еще один способ решения, снова обратимся к общему виду квадратного уравнения . Поскольку , можем разделить обе части уравнения на :

 

Для удобства введем новые обозначения:

Получим:

Теперь в уравнении коэффициент при  равен . Квадратное уравнение в таком виде называют приведенным квадратным уравнением.

В рассмотренных ранее методах решения мы убедились, что если мы можем разложить левую часть уравнения на множители, то уравнение имеет корни.

Верно и обратное утверждение: если данное уравнение имеет корни, то его левую часть можно разложить на множители. Причем, это разложение будет иметь вид:

,

где  и  – это корни уравнения.

Из этого утверждения мы получаем два важных следствия.

Следствие 1. Мы получили еще один способ разложения на множители: многочлен вида  можно представить в виде , где  и  – корни уравнения .

Чтобы получить второе следствие, раскроем скобки в левой части равенства:

Сравнивая коэффициенты в левой и правой части, получаем:

Или:

Эти соотношения являются записью теоремы Виета. Итак, мы получили еще один способ решения квадратного уравнения: если подобрать такие числа  и , что:

то они будут корнями приведенного квадратного уравнения .

 

Пример 8. Решить уравнение:

Решение.

Составим приведенное квадратное уравнение:

По теореме Виета:

Подберем такие числа, которые удовлетворяют этим условиям. Этими числами являются  и , ведь:

Таким образом:

Ответ: .

С несколькими рекомендациями о том, как быстро подбирать корни по теореме Виета, вы можете ознакомиться ниже.


 

Подбор корней

Несколько рекомендаций по подбору корней.

1. Лучше всего подбор начинать с произведения корней. Раскладываете свободный член на множители и проверяете, выполняется ли соотношение для суммы корней:

 можно разложить на  и ; сумма будет  – не подходит. Можно на  и ; сумма равна  – подходит:

2. Подбирая корни, можно сначала подобрать модули корней, а затем уже определиться с их знаками. Рассмотрим на примере:

Знак перед свободным членом «плюс»,  значит, корни одного знака. Поэтому раскладываем на множители  так, чтобы сумма была . Подходят  и . Теперь определяемся со знаками. Сумма корней равна , значит, числа берем со знаком «минус»:

Еще пример:

Знак перед свободным членом «минус»,  значит, корни разного знака. Поэтому раскладываем на множители  так, чтобы разность была равна . Подходят  и . Теперь определяемся со знаками. Сумма корней равна , значит,  должно быть со знаком «плюс», а  – со знаком «минус»:

3. Теорему Виета удобно применять к приведенному уравнению с целочисленными коэффициентами. Но если изначально в уравнении коэффициент , то могут возникнуть дробные числа, с которыми работать менее удобно:


Этого можно избежать, рассмотрев другое приведенное уравнение:



Сделав замену , получаем приведенное квадратное уравнение:

Задача свелась к нахождению таких корней уравнения , произведение которых равно, а сумма равна . Корни исходного уравнения будут в  раз меньше:

Рассмотрим на примере уравнения:

Ищем числа, произведение которых равно , а сумма равна . Это числа  и . Корни исходного уравнения в  раза меньше этих чисел, т.е.


Разберем несколько типичных заданий, в которых удобно использовать теорему Виета.

 

Задание 1. Один из корней уравнения  равен . Определить второй корень уравнения.

Решение.

Составим приведенное квадратное уравнение:

По теореме Виета:

Один из корней равен единице , тогда:

Ответ: .

 

Задание 2. Найти значение выражения:

,

где  и  – корни уравнения:

Решение.

Конечно, можно найти корни уравнения с помощью дискриминанта и затем вычислить сумму кубов. Но вы можете убедиться, что корни не будут целыми числами, поэтому расчеты будут затруднительны. Данное задание удобнее решать с помощью теоремы Виета:

Выразим искомое выражение через сумму и произведение корней. Используем формулу суммы кубов :

Можем подставить значения из теоремы Виета:

Осталось выразить . Для этого выделим полный квадрат:

Подставляем значения из теоремы Виета:

Таким образом:

Ответ: .

 

Заключение

Мы рассмотрели несколько способов решения квадратных уравнений.

  1. Разложение на множители. Этим методом быстро и удобно решать неполные квадратные уравнения.
  2. Использование готовых формул для корней квадратного уравнения. Этот алгоритм является наиболее универсальным и четким: можно решить любое квадратное уравнение и есть строгий алгоритм действий.
  3. Использование теоремы Виета. Позволяет подбирать корни квадратного уравнения. Способ удобен для уравнений, корни которого являются целыми числами.

Какой способ лучше? Попробуйте сами и выберите наиболее удобный для себя: кому-то легче угадывать по теореме Виета, а кто-то будет четко идти по алгоритму, считая дискриминант. Различие понятно – подобрать корни по теореме Виета можно не всегда, и здесь больше элемент везения, а считать через дискриминант – это всегда наверняка, но дольше.

Итак, вы уже знаете алгоритмы решения линейных и квадратных уравнений. Естественно, при моделировании различных задач могут встретиться и более сложные уравнения. Некоторые из них можно свести к решению линейных и квадратных уравнений.

С тем, как это можно сделать, вы познакомитесь на практическом занятии. В общем виде также можно выписать формулы для решения любого уравнения 3 и 4 степени. Но эти алгоритмы решения не входят в курс школьной алгебры, поскольку они достаточно сложные и требуют введения понятия комплексного числа.

Интересна ситуация с уравнениями 5 и выше степени. Есть теорема о существовании корней этих уравнений. А другая теорема утверждает, что не существует алгоритмов, позволяющих найти точные решения этих уравнений в общем виде. Для уравнений 5 и выше степеней решения можно найти только для некоторых уравнений или же найти корни любого, но приближенно. Вот такая вот ситуация: решения есть, а универсальной формулы для их нахождения нет.

 

Список рекомендованной литературы.

  1. Никольский С.М., Решетников Н.Н., Потапов М.К., Шевкин А.В. Алгебра. 8 класс. Учебник. ФГОС, издательство «Просвещение», 2018.
  2. Дорофеев Г.В., Суворова С.Б., Бунимович Е.А. Алгебра. 8  класс. Учебник, издательство «Просвещение», 2018.
  3. Макарычев Ю.Н., Миндюк Н.Г., Нешков К.И., Суворова С.Б./Под ред. Теляковского С.А. Алгебра. 8 класс. Учебник, издательство «Просвещение», 2018

 

Рекомендованные ссылки на ресурсы интернет.

  1. Интернет-портал «youclever.org»
  2. Интернет-портал «school-assistant.ru»
  3. Интернет-портал «yaklass.ru»

 

Домашнее задание.

1. Решить уравнение:

2. Уравнение  имеет два корня  и . Найти:

3. В уравнении  один из корней равен . Найти второй корень и коэффициент .

 

Окружности Карлейля, или Как решать квадратные уравнения циркулем и линейкой

Константин Кноп
«Квант» №7, 2020

Казалось бы, какая может быть связь между квадратными уравнениями и геометрическими инструментами — циркулем и линейкой? Однако эта связь есть, и ее исследование с разных сторон — увлекательнейшая задача, тесно связанная и с геометрией, и с алгеброй.

А при чем тут Томас Карлейль? Согласно Википедии, это британский писатель, публицист, историк и философ, автор многотомных сочинений «Французская революция» (1837), «Герои, почитание героев и героическое в истории» (1841), «История жизни Фридриха II Прусского» (1858–1865). О его занятиях математикой практически ничего не известно, и тем не менее, в честь него назван класс окружностей, помогающих решать квадратные уравнения.

А теперь — обо всем по порядку.

Что мы подразумеваем под решением уравнений циркулем и линейкой?

В школьных учебниках геометрии есть какие-то упоминания о возможности построения квадратичных иррациональностей с помощью геометрических инструментов, но в основном не в виде практических алгоритмов построения, а в качестве теоремы существования. Единственный школьный пример конкретного алгоритма, как-то связанного с квадратными уравнениями, — построение среднего геометрического. Давайте и мы тоже начнем с этой задачи.

Задача 1. Даны отрезки

\(a\) и \(b\). 2\).)

Обычно после решения этой задачи в школе говорится, что поскольку мы умеем с помощью циркуля и линейки складывать, вычитать, также умножать и делить (пользуясь теоремой Фалеса), а вот теперь умеем и корни извлекать, то этого уже достаточно для того, чтобы по формулам для корней квадратного уравнения научиться строить отрезки соответствующей длины. Действительно, достаточно. Но ведь дико неудобно!

К счастью, есть и нормальный способ «геометрического» решения квадратных уравнений, известный вот уже более двух веков. Этот способ впервые был описан шотландским математиком и физиком Джоном Лесли в его книге «Элементы геометрии и плоской тригонометрии», изданной в 1809 году. В одном из последующих изданий Лесли добавил сноску о том, что идея этого способа принадлежит его бывшему ученику Томасу Карлейлю.

На рисунке 2, заимствованном из книги Лесли, сформулирована задача: «Построить прямоугольник, равновеликий данному прямоугольнику \(CDFE\), полупериметр которого равен данному отрезку \(BC\)».

Ее решение, предложенное Карлейлем, показано на рисунке 3. Прокомментируем построение на этом рисунке. Вначале данный отрезок \(BC\) откладывается на продолжении меньшей стороны четырехугольника \((CE)\). Затем на большей его стороне откладывается \(CG=CE\), после чего с помощью параллельных прямых достраивается четвертая вершина прямоугольника \(ABCG\). Теперь достаточно построить окружность на диаметре \(AD\) — точки \(J\) и \(K\) ее пересечения с отрезком \(BC\) и дадут искомые стороны прямоугольника (\(BJ\) и \(JC\) или, как изображено на рисунке, \(BK\) и \(KC=KI\)).

Почему и как это работает? Проще всего ответить на этот вопрос с помощью системы координат. Поместим начало координат в точку \(C\), ось абсцисс направим влево (к \(B\)), а ось ординат вверх (к \(D\)). Кроме того, будем считать, что \(CE=e\), \(CB=b\), \(CD=d\). Тогда координатами точки \(A\) будут \((b;\ e)\), а значит, координатами \(M\) будут \((b/2;\ (d+e)/2)\). Это означает, что окружность с центром \(M\) и радиусом \(MG\ (=MA=MD)\) имеет уравнение

\[\left(x-\frac b2\right)^2+\left(y-\frac{d+e}{2}\right)^2=\left(\frac b2\right)^2+\left(\frac{d-e}{2}\right)^2,\]

откуда

\[x^2-bx+\left(\frac b2\right)^2+y^2-(d+e)y+\left(\frac {d+e}2\right)^2=\left(\frac b2\right)^2+\left(\frac{d-e}{2}\right)^2\]

и

\[x^2-bx+y^2-(d+e)y+de=0. 2-bx+de=0.\]

Как мы знаем из школьной алгебры (теорема Виета), сумма корней этого уравнения равна \(b\), а произведение равно \(de\). Таким образом, если \(K\ (x_1;\ 0)\) и \(J\ (x_2;\ 0)\), то \(CK=CJ=x_1+x_2=b=CB\), а \(CK\cdot CJ=x_1\cdot x_2=de\), т. е. площадь прямоугольника со сторонами \(CK\) и \(BK=CJ\) действительно равна площади исходного прямоугольника \(CDFE\).

То же самое можно было получить и не выписывая уравнения окружности, из чисто геометрических соображений. Ведь \(CK\cdot CJ\) — это произведение длины секущей на ее внешнюю часть, a \(de=CD\cdot CG\) — другое такое же произведение. По теореме о секущих, они равны, т. е. \(CK\cdot CJ=de\). А так как \(ABCD\) — прямоугольная трапеция, а \(M\) — середина ее боковой стороны, то высота в равнобедренном треугольнике \(MJK\) является одновременно средней линией трапеции и медианой в том же треугольнике. Отсюда сразу получаем, что \(CK=BJ\), а значит, \(CK+CJ=CB=b\).

Осмысление того, что способ Карлейля годится не только для одной этой задачи, пришло далеко не сразу. 2-sx+p=0\). Тогда мы должны взять на координатной плоскости точки \(A\ (0;\ 1)\) и \(B\ (s;\ p)\), найти середину \(C\) отрезка \(AB\) и построить окружность с центром в \(C\) и радиусом \(CA\) (рис. 4). Точки пересечения этой окружности с осью абсцисс и будут корнями уравнения.

Доказательство практически полностью повторяет решение задачи Лесли, приведенное выше.

Но мы, кажется, собирались что-то строить циркулем и линейкой? А здесь координатная плоскость… Ничего страшного! Все, что нам на самом деле нужно от координат, вполне сводится к двум осям. Действительно, вместо точки \(B\ (s;\ p)\) можно сразу рассмотреть ее проекции на оси \(B_1\ (s;\ 0)\) и \(B_2\ (0;\ p)\). Ведь если мы знаем коэффициенты \(a\) и \(p\), то построить именно эти точки на осях — проще простого. А дальше центр окружности получится пересечением серединных перпендикуляров к \(OB_1\) и \(AB_2\) (постарайтесь разобраться, почему это так, по рисунку 5).

Как видно из рисунка 5, способ с окружностью Карлейля не только простой, но и удивительно легкий для запоминания. Кроме начала координат, требуются всего три точки — \(B_1\) (абсцисса которой равна коэффициенту \(s\)) откладывается на оси абсцисс, а \(A\ (0;\ 1)\) и \(B_2\) (ордината которой равна \(p\)) — на оси ординат. Два серединных перпендикуляра, одна окружность — вуаля, корни готовы!

Как это применять?

Возможно, самое известное применение окружностей Карлейля — построение правильных многоугольников. Многие слышали о том, что построение правильного пятиугольника циркулем и линейкой возможно. (На самом деле — не только возможно, но было дано и полностью обосновано еще Евклидом.) Но многие ли пытались понять, как именно решается эта задача?

Задача 2. Дана окружность с центром \(O\) и точка \(A\) на ней. Постройте правильный пятиугольник \(ABCDE\), вписанный в эту окружность.

Решение. Построение пятиугольника с помощью окружностей Карлейля отличается удивительной простотой. Начнем с координатных осей, проведя через центр круга прямую \(OX\), перпендикулярную \(OA\) (рис. {\circ})\), т. е. углы \(AOB\) и \(AOE\) равны 72 градусам, а углы \(AOC\) и \(AOD\) — 144 градусам. А это и означает, что они являются вершинами правильного пятиугольника. Доказательство правильности построения на этом завершено.

О сложности построений и «геометрографии» Эмиля Лемуана

Француз Эмиль Мишель Гиацинт Лемуан по праву считается одним из отцов современной «геометрии треугольника». Вероятно, вы слышали о точке Лемуана в треугольнике или об окружностях Лемуана. Он же в 1892 году опубликовал работу «Геометрография, или искусство геометрических конструкций» («La Géométrographie ou l’art des constructions géométriques»), в которой впервые предложил сравнивать различные геометрические построения по количеству требуемых элементарных операций (а лучшим построением считать то, на которое тратится меньшее число операций). Оптимальным («геометрографическим») способом Лемуан называет самый экономный из известных алгоритмов построения.

Элементарных операций Лемуан насчитал пять:
    \(S_1\) — приложить линейку к определенной точке;
    \(S_2\) — провести прямую линию;
    \(C_1\) — поставить ножку циркуля в определенную точку;
    \(C_2\) — изменить радиус циркуля, поставив ножку с карандашом в определенную точку;
    \(C_3\) — провести циркулем окружность.

При этом Лемуан считал циркуль устойчивым, т. е. после проведения окружности сохраняющим выставленный радиус. Таким образом, следующая окружность того же радиуса, по Лемуану, требует уже не трех операций: \(C_1+C_2+C_3\), а всего двух: \(C_1+C_3\).

Сосчитаем, например, количество операций в построении корней уравнения с помощью окружности Карлейля, показанном на рисунке 5. Оси координат и точки \(A\), \(B_1\) и \(B_2\) заданы. Построение серединного перпендикуляра к отрезку \(OB_1\) — это две окружности равных радиусов и одна прямая, т. е. \((2C_1+C_2+2C_3)+(2S_1+S_2)\). Следующий серединный перпендикуляр — это \((2C_1+2C_3)+(2S_1+S_2)\), потому что мы строим его, не меняя радиуса циркуля. Тем самым, мы построили центр, затратив 15 элементарных операций. И наконец, сама окружность Карлейля — это еще \(C_1+C_2+C_3\). Итого «сложность» построения \(5C_1+2C_2+5C_3+4S_1+2S_2\) равна 18.

Упражнение 1. Придумайте другой способ построения окружности Карлейля, имеющий сложность 17 или меньше.
Подсказка. Центр этой окружности — середина отрезка, одним из концов которого является \(B_1\). Постройте сначала второй конец этого отрезка.

Современная компьютерная игра-головоломка Euclidea (euclidea.xyz) также требует от решателей задач найти самое экономное построение, но, в отличие от Лемуана, не подсчитывает предварительные операции \(S_1\), \(C_1\) и \(C_2\), а считает только количество проведенных линий (т. е. \(S_2+C_3\) в терминологии Лемуана). С точки зрения Euclidea, построение окружности Карлейля имеет сложность 7.

Можно ли отыскать корни квадратного уравнения с меньшей сложностью?

Когда автор начинал писать эту статью, он предполагал, что этот вопрос будет повешен в качестве финальной улыбки Чеширского Кота и оставлен читателю для самостоятельного решения. Однако планы немножко поменялись, и сейчас я хочу показать более экономный способ, а читателям предложить обдумать, нельзя ли найти еще лучший.

Во-первых, откажемся от оси ординат. Зачем она нужна, если все корни мы ищем на оси абсцисс?

Пусть у нас есть всего одна ось, а на ней — четыре известные точки \(O\ (0)\), \(E\ (1)\), \(S\ (X+Y)\) и \(P\ (XY)\). Последние две точки соответствуют коэффициентам того уравнения, которое мы хотим решить. Рисунок 7 соответствует ситуации, когда \(0<XY<1<X+Y\), но это не очень критично — приведенное ниже построение работает и во многих других случаях, а если оно не будет работать из-за того, что какие-то окружности не пересекутся, то его не очень сложно адаптировать.

Вначале проведем две окружности равных радиусов с центрами в \(P\ (XY)\) и \(E\ (1)\) (рис. 8) — как будто мы хотим строить середину отрезка между этими точками. Радиус окружностей возьмем таким, чтобы первая окружность прошла через точку \(S\ (X+Y)\). Отметим \(C\) — общую точку этих окружностей.

Затем построим еще две окружности — окружность с центром \(O\ (0)\), проходящую через \(C\), и окружность такого же радиуса с центром \(S\). Их точку пересечения (любую из двух точек) назовем \(D\) (рис. 9).

И (немного неожиданный, как и полагается улыбке Кота) финальный шаг — окружность с центром \(D\), радиус которой точно такой же, как у двух первых (рис. 2\). А поскольку середина этого отрезка имеет координату \((X+Y)/2\), то правый конец отрезка совпадает с большим из чисел \(X\), \(Y\), а левый — с меньшим.

Сосчитаем сложность: \(2C_1+C_2+2C_3\) на первую пару окружностей, столько же на вторую и еще \(C_1+C_2+2C_3\) на последнюю окружность — всего \(5+5+3=13\), вместо прежних 17 или 18. Ура!

Дополнение.

Построение правильного 17-угольника

Задача 3. Пусть дана окружность с центром \(O\) и точка \(A\) на ней. Постройте правильный 17-угольник с вершиной \(A\), вписанный в эту окружность.

Эта задача, в отличие от трех неразрешимых задач древности, стала знаменитой после того, как была решена 19-летним Карлом Фридрихом Гауссом. Биографы Гаусса пишут, что только после ее решения Гаусс окончательно выбрал своей будущей профессией математику, а не литературу. Известна также легенда, что Гаусс завещал выбить на своем могильном камне изображение правильного 17-угольника. {16}=2\cos(\pi/17)\), так что, зная величину \(\eta_{08}\), мы легко построим вершины правильного 17-угольника, соседние с заданной вершиной \(A\) (аналогично построению вершин по точке \(H_2\) для правильного пятиугольника).

Упражнения

2. Вычислите произведение \(\eta_{08}\eta_{48}\).
3. Дуайн ДиТемпл описал пошаговое построение правильного 17-угольника с помощью окружностей Карлейля (commons.wikimedia.org). Разберитесь в этом построении.

РАЗЛИЧНЫЕ СПОСОБЫ РЕШЕНИЯ КВАДРАТНЫХ УРАВНЕНИЙ

  • Авторы
  • Руководители
  • Файлы работы
  • Наградные документы

Альдебенева А.Н. 1


1ГБОУ СОШ ж.-д.ст. Погрузная Самарской области

Степанова Г.А. 1


1ГБОУ СОЩ ж.-д.ст. Погрузная

Автор работы награжден дипломом победителя III степени

Диплом школьникаСвидетельство руководителя

Текст работы размещён без изображений и формул.
Полная версия работы доступна во вкладке «Файлы работы» в формате PDF

Введение

Математическое образование, получаемое в общеобразовательной школе, является важнейшим компонентом общего образования и общей культуры современного человека. Практически все, что окружает современного человека так или иначе связано с математикой. Поэтому решение многих практических задач сводится к решению различных видов квадратных уравнений, которые необходимо научиться решать.

В данной работе я попыталась обобщить и систематизировать изученный материал по выше указанной теме. Я расположила материал по степени его сложности, начиная с самого простого. В него вошли как известные мне способы решения квадратных уравнений из школьного курса алгебры, так и нестандартные нетрадиционные способы решения, которые не изучаются в школьном курсе, но знание которых сокращает процесс решения квадратных уравнений. Квадратные уравнения – это фундамент, на котором покоится величественной здание алгебры.

Для выявления актуальности темы моей работы я провела исследование среди учащихся 8-11 классов нашей школы. Им было предложено решить полное квадратное уравнение любым известным способом. В исследовании приняло участие 72 учащихся из 86 (84 %) .

Метод решение квадратного уравнения

Кол-во учащихся

1.Метод выделения квадрата двучлена

0 чел

0%

2. Метод разложения левой части уравнения на множители способом группировки

0 чел

0%

3.Решение уравнений по формулам дискриминанта и корней квадратного уравнения

40 чел

56%

4.Решение уравнений, используя теорему Виета

4 чел

15%

5. Решение уравнений графическим способом

0 чел

0%

6.Неверно решили уравнения

8 чел

30%

7.Решение уравнений с использованием свойств коэффициентов

20 чел

27%

Таким образом, ясно, что при решении квадратных уравнений учащиеся нашей школы используют традиционно формулы дискриминанта и корней уравнения, что требует громоздких вычислений и как следствие больших затрат времени, что непозволительно в процессе сдачи экзаменов.

Проблемный вопрос: существуют ли кроме общепринятых приемов решения квадратных уравнений другие, которые позволяют быстро и рационально решать квадратные уравнения? Какие существуют рациональные способы решения квадратных уравнений?

Гипотеза: установление связи между коэффициентами и корнями квадратного уравнения позволит найти эффективные приемы быстрого решения квадратного уравнения.

Цель: установив связь между коэффициентами и корнями квадратного уравнения, найти новые рациональные приемы решения уравнений.

Задачи:

1.Изучить литературу по истории приемов решения квадратных уравнений

2. Обобщить накопленные знания о квадратных уравнениях и способах их решения.

3. Установить зависимость корней квадратного уравнения от его коэффициентов и найти эффективные приемы быстрого решения квадратного уравнения, в том числе с большими коэффициентами.

4.Изложить наиболее известные способы решения квадратных уравнений.

3.Показать нестандартные способы решения квадратных уравнений. Сделать выводы.

5. Разработать дидактический материал для проведения практикума по решению квадратных уравнений с использованием новых приемов в помощь ученикам, увлеченным математикой.

Объект исследования: квадратные уравнения

Предмет исследования: методы и приемы решения квадратных уравнений, в том числе с большими коэффициентами

Актуальность темы: тема «Квадратные уравнения» является одной из самых актуальных. Она находит широкое применение в разных разделах математики, имеет теоретическую и практическую значимость. Ведь почти все, что окружает человека так или иначе связано с математикой. Поэтому решение многих практических задач сводится к решению различных видов уравнений, которые необходимо научиться решать.

Тема исследования:

Нетрадиционные способы решения квадратных уравнений.

Методы исследования: анкетирование, сбор статистических данных, обработка собранных сведений и информации, оформление результатов исследования.

Итог работы.

Каждый ученик должен прийти к выводу «Мой способ решения квадратного уравнения – понятный, но я хочу найти для себя самый рациональный»

Глава 1. Историческая справка.

В те далекие времена, когда мудрецы впервые стали задумываться о равенствах, содержащих неизвестные величины, наверное, еще не было ни монет, ни кошельков. Но зато были кучи, а также горшки, корзины, которые прекрасно подходили на роль тайников-хранилищ, вмещающих неизвестное количество предметов. «Ищется куча, которая вместе с двумя третями ее, половиной и одной седьмой составляет 37…», — поучал во II тысячелетии до новой эры египетский писец Ахмес. В древних математических задачах Междуречья, Индии, Китая, Греции неизвестные величины выражали число павлинов в саду, количество быков в стаде, совокупность вещей, учитываемых при разделе имущества. Хорошо обученные науке счета писцы, чиновники и посвященные в тайные знания жрецы довольно успешно справлялись с такими задачами. Дошедшие до нас источники свидетельствуют, что древние ученые владели какими-то общими приемами решения задач с неизвестными величинами. Однако ни в одном папирусе, ни в одной глиняной табличке не дано описания этих приемов. Авторы лишь изредка снабжали свои числовые выкладки скупыми комментариями типа: «Смотри!», «Делай так!», «Ты правильно нашел». В этом смысле исключением является «Арифметика» греческого математика Диофанта Александрийского (III в.) – собрание задач на составление уравнений с систематическим изложением их решений.

Однако первым руководством по решению задач, получившим широкую известность, стал труд багдадского ученого IX в. Мухаммеда бен Мусы аль-Хорезми. Слово «аль-джебр» из арабского названия этого трактата – «Китаб аль-джебер валь-мукабала» («Книга о восстановлении и противопоставлении») – со временем превратилось в хорошо знакомое всем слово «алгебра», а само сочинение аль-Хорезми послужило отправной точкой в становлении науки о решении уравнений.

Основной материал, связанный с изучением темы «Квадратные уравнения» находится в УМК под редакцией С. А. Теляковского за 8 класс. В учебнике разобраны все основные вопросы по теме:

1. Определение и виды квадратных уравнений

2. Основные методы решения квадратных уравнений

Однако, дополнительный материал, связанный с историей вопроса о возникновении квадратных уравнений можно найти в «Энциклопедия по математике» «Занимательная математика», М., 2007. Способы решения задач на квадратные уравнения в полном объёме раскрыты в изданиях «Сборник элективных курсов» Волгоград, 2006 г. Рациональные приемы решения квадратных уравнений в полном объеме освещены на сайтах интернет.

Таким образом, изученная литература позволила приобрести новые интересные знания по истории возникновения квадратного уравнения, приобрести опыт по решению различных квадратных уравнений и перейти к следующему этапу в исследовании – перенести полученные знания в нестандартную ситуацию.

Глава 2.Обобщение имеющихся знаний о квадратных уравнениях и способах их решения

2.1. Определение квадратного уравнения

Определение: Квадратным уравнением называется уравнение вида

аx 2 + bx + c = 0, где х – переменная, а, b и с– некоторые числа, причем, а ≠ 0.

Числа а, b и с — коэффициенты квадратного уравнения. Число а называют первым коэффициентом, число b– вторым коэффициентом и число c – свободным членом.

● Пример. 8x2 – 7x + 3 = 0

В каждом из уравнений вида ax 2 + bx + c = 0, где а ≠ 0, наибольшая степень переменной x – квадрат. Отсюда и название: квадратное уравнение.

Квадратное уравнение, в котором коэффициент при х 2 равен 1, называют приведенным квадратным уравнением.

● Пример. х 2 – 11х+30=0, х2 – 8х= 0.

2.2. Решение квадратных уравнений с помощью выделения квадрата двучлена

Рассмотрим на примере решение квадратного уравнения, в котором оба коэффициента при неизвестных и свободный член отличны от нуля. Такой способ решения квадратного уравнения называют выделением квадрата двучлена.

Пример. Рассмотрим уравнение 7х 2 – 6х – 1= 0.

Разделив обе части этого уравнения на 7, получим равносильное ему приведенное квадратное уравнение

Х2 – х – = 0.

Выделим из трехчлена х2 – x- –квадрат двучлена. Для этого разность

х2 – х представим в виде х 2 – 2· х, прибавим к ней выражение и вычтем его. Получим

х2 – 2· х + – – = 0.

Отсюда х 2 – 2· х + = + ,

= .

Следовательно, х — = – или х — = , ,

х – = — или х — = ,

х = – или х = 1.

Уравнение имеет два корня: – и 1.

2.3. Решение квадратных уравнений по формуле

Решение квадратных уравнений выделением квадрата двучлена часто приводит к громоздким преобразованиям. Поэтому поступают иначе. Решают уравнение в общем виде и в результате получают формулу корней. Затем эту формулу применяют при решении любого квадратного уравнения. Решим квадратное уравнение

аx 2 + bx + c = 0.

Разделив его обе части на а, получим равносильное ему приведенное квадратное уравнение

х 2 + х + = 0.

Выделим из трехчлена х 2 + х + квадрат двучлена. Для этого сумму

х 2 + х представим в виде х 2 +2х∙ ,прибавим к ней выражение

и вычтем его. Получим

х2 +2х∙ + – + = 0,

х 2+2х∙ + = – ,

= – ,

= ,

Уравнение = равносильно уравнению ax 2 + bx + c = 0.

Число его корней зависит от знака дроби . Так как а ≠ 0, то 4a–положительное число, поэтому знак этой дроби определяется знаком его числителя, т. е. выражения b2 – 4ас. Это выражение называют дискриминантом квадратного уравнения ax 2 + bx + c = 0. Его обозначают буквой D, т.е.

D = b – 4ас. Дискриминант квадратного уравнения ax 2 + bx + c = 0

– выражение b2 – 4ас= D – по знаку которого судят о наличии у этого уравнения действительных корней.

Различные возможные случаи в зависимости от значения D.

1.Если D>0, то уравнение имеет два корня:

X1 =и x2 =

Пример. Рассмотрим уравнение 2x 2 –3x + 1= 0.

а=2; b= –3; с=1,

D= b – 4ас =(–3) – 4ас= 9–8= 1; 2 корня.

X1= = = = 0,5

X2 = = = =1

Ответ: 0,5;1

2.Если D= 0, то уравнение имеет один корень:

х = – .

Пример. Рассмотрим уравнение 9х2 +6х+1= 0.

а=9; b= 6; с=1,

D= b – 4ас=6 – 4ас=36–36= 0; 1 корень.

X= = = -0,3

Ответ: -0,3

3. Если D 0. тогда это уравнение имеет два корня:

x1 =и x2 =

Найдем сумму и произведение корней:

x1 + x 2= + = = –p;

x1 . x 2 = . = = = =q.

Следовательно,

x1 + x 2 =-p, x1 . x 2 =q .

Пример. Рассмотрим уравнение х 2 – 3х + 2 = 0.

D =1, уравнение имеет два корня. х1 = 2 и х2 = 1, p= –3; q= 2.

По теореме Виета x1 + x 2 =-p , значит 2 + 1= 3;

x1 . x 2 =q , значит 2 ∙ 1 =2.

Следовательно, х 1 = 2 и х2 = 1 являются корнями уравнения х2 – 3х + 2 = 0.

При D = 0 корни уравнения можно вычислить по формуле

х = и x= .

Квадратное уравнение ax 2 + bx + c = 0 имеет корни х1 и х 2 равносильное ему приведенное квадратное уравнение имеет вид

х + х = – , х ∙ х = .

Справедливо утверждение, обратное теореме Виета:

Теорема: Если числа m и n таковы, что их сумма равна –p, а произведение

равно q, то эти числа являются корнями уравнения х 2 + px + q = 0.

Пример. Рассмотрим уравнение х 2 +3х – 40=0.

D= 32+4 ∙40= 169.

По формуле корней квадратного уравнения получаем

х = ; х = .

Отсюда х 1 =-8 ; х 2 =5.

Покажем, что корни уравнения найдены правильно. В уравнении

х2 +3х – 40=0 коэффициент р равен 3, а свободный член q равен –40. Сумма найденных чисел –8 и 5 равна –3, а их произведение равно –40. Значит, по теореме, обратной теореме Виета, эти числа являются корнями уравнения

х2 +3х – 40=0.

Итак, квадратные уравнения — это фундамент, на котором покоится величественное здание алгебры. В школьном курсе математики изучаются формулы корней квадратных уравнений, с помощью которых можно решать любые квадратные уравнения. Однако имеются и другие способы решения квадратных уравнений, которые позволяют очень быстро и рационально решать уравнения. Имеется десять способов решения квадратных уравнений. Подробно в своей работе я разобрала некоторые из них, которые сама очень активно применяю.

Глава 3. Рациональные способы решения квадратного уравнения.

3.1.Свойства коэффициентов квадратного уравнения.

1) Если а+ b+c= 0, х1 = 1 , х 2 =

Пример. Рассмотрим уравнение х2 +4х – 5= 0.

a+ b+c= 0, х 1 =1 , х 2 = . 1+4+ (-5) =0 .

Значит, корнями этого уравнения являются 1 и –5. Проверим это с помощью нахождения дискриминанта:

D= b – 4ас= 4 – 4∙1∙(–5)= 36.

х = = = – 5.

х = = =1.

Отсюда следует, что если а+b+c= 0 , то х 1 =1 , х 2 =

2) Если b= а+c, то х1 =-1, х 2 = -

Пример. Рассмотрим уравнение 2х2 +8х +6 = 0.

2) Если b= а+c, то х1 =-1, х 2 = — . 8= 2+6

Значит корнями этого уравнения являются –1 и –3. Проверим это с помощью нахождения дискриминанта:

D= b – 4ас=8 – 4∙2∙6= 16.

х = = = –3.

х = = = –1.

Отсюда следует, что если b= а+c , то х1 = -1 , х 2 =

Пример 345х2 – 137х – 208 = 0.

Решение. Так как а + b + с = 0 (345 – 137 – 208 = 0), то

х1 = 1, х2 = c/a = -208/345.

Ответ: 1; -208/345.

Пример 132х2 – 247х + 115 = 0.

Решение. Так как а + b + с = 0 (132 – 247 + 115 = 0), то

х1 = 1, х2 = c/a = 115/132.

Ответ: 1; 115/132

3). Если второй коэффициент b = 2k – четное число, то формулу корней

Пример. Решим уравнение 3х2 — 14х + 16 = 0.

Решение. Имеем: а = 3, b = — 14, с = 16, k = — 7;

D = k2 – ac = (- 7)2 – 3 • 16 = 49 – 48 = 1, D > 0, два различных корня;

Ответ: 2; 8/3

3.2. Способ «переброски».

При этом способе коэффициент а умножается на свободный член, как бы «перебрасывается» к нему, поэтому его и называют способом «переброски». Этот способ применяют, когда можно легко найти корни уравнения, используя теорему Виета и, что самое важное, когда дискриминант есть точный квадрат.

Если а±b+c≠0, то используется прием переброски:

  1. 2 – 11х+5=0 х2 – 11х+10= 0

х = 10; х =1. Корни уравнения необходимо поделить на 2.

Ответ: 5; 0,5.

  1. 2 -37 х +9 =0 Ответ: ¼, 9

3.3.Закономерность коэффициентов

1) Если в уравнении ax 2+ bx + c = 0 коэффициент b равен (а2 +1), а коэффициент с численно равен коэффициенту а, то его корни равны

х = –а; х = – .

ах 2 + (а2 +1)∙ х+ а= 0

Пример. Рассмотрим уравнение 6х2 +37х +6 = 0.

х = –6; х = – .

2) Если в уравнении ax2 – bx + c = 0 коэффициент b равен (а2 +1),а коэффициент с численно равен коэффициенту а, то его корни равны

х = а; х = .

ax 2– (а2 +1)∙ х+ а= 0

Пример. Рассмотрим уравнение 15х 2–226х +15 = 0.

х = 15; х = – .

3) Если в уравнении ax2 + bx – c = 0 коэффициент b равен (а2 –1), а коэффициент с численно равен коэффициенту а, то его корни равны

х = –а; х = .

ax2 + (а2 –1)∙ х– а= 0

Пример. Рассмотрим уравнение 17х2 +288х – 17 = 0.

х = –17; х = .

4) Если в уравнении ax 2– bx – c = 0 коэффициент b равен (а2 –1), а коэффициент с численно равен коэффициенту а, то его корни равны

х = а; х = – .

ax 2+ (а2–1)∙ х– а= 0

Пример. Рассмотрим уравнение 10х2–99х – 10 = 0.

х = 10; х = – .

3.4.Графическое решение квадратного уравнения.

Если в уравнении

х2 + px + q = 0

перенести второй и третий члены в правую часть, то получим

х2 = — px — q.

Построим графики зависимости у = х2 и у = — px — q.

График первой зависимости — парабола, проходящая через начало координат. График второй зависимости —

прямая (рис.1). Возможны следующие случаи:

— прямая и парабола могут пересекаться в двух точках,

абсциссы точек пересечения являются корнями квад- ратного уравнения;

— прямая и парабола могут касаться (только одна общая точка), т.е. уравнение имеет одно решение;

— прямая и парабола не имеют общих точек, т.е. квадратное уравнение не имеет корней.

Примеры.

1) Решим графически уравнение х2 — 3х — 4 = 0 (рис. 2).

Решение. Запишем уравнение в виде х2 = 3х + 4.

Построим параболу у = х2 и прямую у = 3х + 4. Прямую

у = 3х + 4 можно построить по двум точкам М (0; 4) и

N (3; 13). Прямая и парабола пересекаются в двух точках

А и В с абсциссами х1 = — 1 и х2 = 4. Ответ: х1 = — 1;

х2 = 4.

2) Решим графически уравнение (рис. 3) х2 — 2х + 1 = 0.

Решение. Запишем уравнение в виде х2 = 2х — 1.

Построим параболу у = х2 и прямую у = 2х — 1.

Прямую у = 2х — 1 построим по двум точкам М (0; — 1)

и N(1/2; 0). Прямая и парабола пересекаются в точке А с

абсциссой х = 1. Ответ: х = 1.

3) Решим графически уравнение х2 — 2х + 5 = 0 (рис. 4).

Решение. Запишем уравнение в виде х2 = 5х — 5. Построим параболу у = х2 и прямую у = 2х — 5. Прямую у = 2х — 5 построим по двум точкам М(0; — 5) и N(2,5; 0). Прямая и парабола не имеют точек пересечения, т.е. данное уравнение корней не имеет.

Ответ. Уравнение х2 — 2х + 5 = 0 корней не имеет.

Итак, квадратным уравнением называется уравнение вида ax2 + bx + c = 0. Квадратные уравнения бывают полными, неполными и приведенными. Способы решений полных уравнений различны: выделение квадрата двучлена, по формуле, по теореме Виета, способ переброски, способы, основанные на свойствах и закономерностях коэффициентов квадратного уравнения. В данной работе я изложила и показала на примерах все эти способы. Проанализировав дополнительный материал, я пришла к выводу, что с помощью рациональных способов решения квадратных уравнений , решать уравнения стало намного намного проще и быстрее.

Предложенные методы решения квадратных уравнений просты в применении, и они, безусловно, должно заинтересовать увлекающихся математикой учеников. Моя работа дает возможность по-другому посмотреть на те задачи, которые ставит перед нами математика.

Заключение.

Таким образом, я считаю, что тема данного исследования полностью раскрыта. При работе над темой я узнала много нового из истории квадратных уравнений, а также научилась их решать более удобным способом. Полученные знания пригодятся мне в будущем.

В процессе работы мною создана система нестандартных приемов решения квадратных уравнений и разработан банк заданий, на основе которого мною проведена успешная апробация этих приемов. Хочется отметить и то, что излагаемая тема в этой работе еще полностью не изучена, она таит в себе много скрытого и неизвестного, что дает прекрасную возможность для дальнейшей работы над ней.

Данный материал можно рекомендовать для внеклассных занятий по математике. Материалом могут воспользоваться те, кто любит математику и хочет знать о математике больше.

Список литературы:

1.Алгебра 8 класс: учебник для общеобразовательных учреждений. Авторы: Ю. Н. Макарычев, Н. Г. Миндюк, К. И. Нешков, С. Б. Суворова. Издательство «Просвещение», Москва 2009 г.

2.Брадис В.М. Четырехзначные математические таблицы: для сред.шк.-57-е изд. – М.: Просвещение, 1990.

3. Штейнгауз В.Г. Математический калейдоскоп. – М.: Бюро «Квантум», 2005.

Просмотров работы: 7218

Введение в задачи с параметром: решение уравнений с параметром

На этой странице вы узнаете
  • Игра в прятки: как значение одной переменной может помочь найти другую?
  • Парадокс: как стоять на месте и бежать с любой скоростью одновременно? 
  • Решаем параметры осторожно: как не совершить ошибку в квадратном уравнении с параметром? 

Мы привыкли, что в уравнении коэффициенты не меняются. Но возможно ли из одного уравнения составить бесконечное множество различных его вариантов? Узнаем об этом в статье. 

Что такое параметр 

Утром на термометре было некоторое количество градусов, которое мы обозначим за х. В обед температура воздуха изменилась в несколько раз. Во сколько раз должна была измениться температура воздуха, чтобы на термометре было 20 градусов? 

Такие задачи достаточно легко решаются. Если бы изначально было пять градусов, то искомое число было бы равно \(\frac{20}{5} = 4\). А если было 10 градусов, то искомое число было бы равно \(\frac{20}{10} = 2\). 

Но не все так просто. Мы не знаем, какой изначально была температура. Также мы не знаем, во сколько раз она изменилась. То есть мы получили уравнение с двумя неизвестными переменными. 

Обозначим вторую переменную a, у нас получится уравнение вида ax=20. Только что введенная нами переменная “a”  называется параметр. 

Параметр — это условная буква, вместо которой можно подставить число.  

То есть параметр — это еще одна переменная, которая может принять несколько значений. 

Как решать уравнения с параметром, если у нас целых две (а то и больше) неизвестных переменных? Нужен иной подход, чем при решении обычного уравнения.

Решить уравнение с параметром — это найти такие числовые значения параметра, при которых условие выполняется. 

Мы ищем не единственное значение параметра, а все возможные его значения для заданного условия.

Игра в прятки: как значение одной переменной может помочь найти другую?

Поскольку параметр — переменная в уравнении, которая является коэффициентом, его значение задает и корни уравнения. То есть переменные а и х зависят друг от друга так же, как и зависят корни обычного уравнения от его коэффициентов. 

Линейные уравнения с параметром

Вернемся к нашей погоде. У нас получилось уравнение ax = 20. Как найти, сколько градусов было изначально? Разделить все уравнение на число a. 

\(x = \frac{20}{a}\)

Какие значения может принимать параметр? Любые. Например, при a = 1 x = 20.
При a = 2 x = 10.
При a = 40 x = 0,5 

Что, если a=0? Мы получаем уравнение \(x = \frac{20}{0}\), у которого нет решения, поскольку на 0 делить нельзя. 

Если мы не будем преобразовывать изначальное уравнение, то получится 0*x=20, то есть уравнение не будет выполняться: какое бы число мы ни умножили на 0, получится 0. 

Получается, решение есть при любых значениях a, кроме 0. Таким образом, мы и нашли ответ: при a = 0 решений нет, при a \(\neq\) 0 — x = 20a. 

Добавим немного теории. Представим наше уравнение в виде ax = b, где a, b — действительные числа. Рассмотрим несколько случаев. 

1) b \(\neq\) 0. 

Предположим, Пете необходимо в несколько раз увеличить скорость х, пробежать дистанцию и поставить рекорд. Чтобы поставить рекорд, он должен бежать со скоростью 15 км/ч — это и будет коэффициент b

Получаем уравнение ax = 15. Как найти начальную скорость Пети? \(x = \frac{15}{a}\). 

Такое уравнение мы уже решали выше. Получаем два случая: 

  • Если a = 0 — решений нет. 
  • Если a \(\neq\) 0, то изначальная скорость Пети была равна \(x = \frac{15}{a}\). 
Парадокс: как стоять на месте и бежать с любой скоростью одновременно?

Когда Пете нужно увеличить скорость в 0 раз, получается парадокс. 
С какой бы скоростью ни бежал Петя, он все равно будет стоять на месте, поскольку 0 * x = 0. Даже если он изначально бегал со скоростью света, его скорость останется равна 0, а не 15 км/ч. 

2) b = 0. 

Мы получаем уравнение ax = 0. Также разберем два случая значений параметра: 

  • a = 0. Мы получаем уравнение 0 * x = 0. Какое значение х нужно подставить, чтобы уравнение выполнялось? 

Какое бы число мы ни умножили на 0, получим 0. Получаем бесконечное множество решений. 

  • a \(\neq\) 0. Здесь получается, что равен 0 уже х: \(x = \frac{0}{a} = 0\). 

Подведем итог. Как можно решить уравнение вида ax = b?

  • Если a = 0, b = 0 — бесконечное множество решений. 
  • Если a = 0, b \(\neq\) 0 — решений нет. 
  • Если a \(\neq\) 0, b \(\neq\) 0 — решением будет \(x = \frac{b}{a}\). 

Квадратные уравнения с параметром

Прежде чем приступать к изучению следующего материала, рекомендуем ознакомиться с понятием квадратного уравнения в статье «Линейные, квадратные и кубические уравнения». Также важно ориентироваться в графиках параболы из статьи «Основные элементарные функции». 

Квадратное уравнение имеет вид ax2 + bx + c = 0, а графиком функции y = ax2 + bx + c будет парабола. 

Как работать с такими уравнениями, если в них присутствует параметр? В первую очередь, важны рассуждения. Любое задание с параметром можно решить, проанализировав функцию.  

Решение квадратного уравнения опирается на понятие дискриминанта. В зависимости от его значений может получиться разное количество корней: 

  • При D > 0 уравнение имеет два корня. 
  • При D = 0 уравнение имеет один корень. 
  • При D < 0 уравнение не имеет корней. 

Как это проверить на графике? Корни уравнения — это точки, в которых парабола пересекает ось абсцисс, то есть ось х

Рассмотрим три уравнения. 

1) x2 — x — 2 = 0
Решим уравнение с помощью дискриминанта. 
D = 12 — 4 * 1 * (-2) = 1 + 8 = 9
Поскольку дискриминант больше 0, то уравнение имеет два корня. 

\(x_1 = \frac{1 + 3}{2} = 2\)
\(x_2 = \frac{1 — 3}{2} = -1\)

Проверим с помощью графика функции. Построим параболу и заметим, что она действительно дважды пересекает ось абсцисс, а координаты этих точек равны (−1; 0) и (2; 0) . 

2) x2 -4x + 4 = 0
Решим уравнение с помощью дискриминанта.  
D = 16 — 4 * 1 * 4 = 16 — 16 = 0
Поскольку дискриминант равен 0, у уравнения всего один корень. 

\(x = \frac{4}{2} = 2\)

Проверим на графике. И действительно, парабола касается оси х только один раз в вершине, координаты которой (2; 0). 

3) x2 — 5x + 7 = 0
Решим уравнение с помощью дискриминанта. 
D = 25 — 4 * 1 * 7 = 25 — 28 = -3

Поскольку дискриминант отрицательный, у уравнения нет корней. И это отлично видно, если посмотреть на график функции: парабола лежит выше оси х и никогда ее не пересечет. 

Где можно применить эти знания, решая параметры? 

Пример 1. Найдите все значения параметра a, при которых уравнение x2 + (3a + 11)x + 18,25 + a = 0 имеет два различных решения. 

Решение. Перед нами квадратное уравнение с коэффициентами b = 3a + 11, c = a + 18,25. В каких случаях это уравнение будет иметь два различных корня?

Квадратное уравнение имеет два корня, если D > 0. Нужно найти все значения параметра, при которых дискриминант будет положительным. 

1. Для начала найдем сам дискриминант. 

D = (3a + 11)2 — 4 * 1 * (a + 18,25) = 9a2 + 66a + 121 — 4a — 73 = 9a2 + 62a + 48

2. Поскольку дискриминант должен быть больше 0, то получаем неравенство 9a2 + 62a + 48 > 0

3. Решим его «Методом интервалов».

9a2 + 62a + 48 = 0
D = 3844 — 1728 = 2116
\(a_1 = \frac{-62 + 46}{18} = -\frac{16}{18} = -89\)
\(a_2 = \frac{-62 — 46}{18} = -\frac{108}{18} = -6\)

4. Дискриминант будет положительным при \(a \in (-\infty; -6) \cup (-\frac{8}{9}; +\infty)\). Это и будет ответ. 

Ответ: \(a \in (-\infty; -6) \cup (-\frac{8}{9}; +\infty)\).

Важно: в уравнении мы указываем не сами решения уравнения, а значения параметра, при которых уравнение имеет два решения. 

Пример 2. При каких значениях параметра a уравнение (2a + 1)x2 — ax + 3a + 1 = 0 имеет два различных решения? 

Решение. Этот пример похож на предыдущий, однако здесь есть одна важная особенность. Что произойдет с уравнением, если 2a+1 = 0? 

Мы получим уравнение 0,5x — 0,5 = 0, то есть линейное уравнение. У уравнения будет всего одно решение, что уже не подходит под условие задачи. 

Решаем параметры осторожно: как не совершить ошибку в квадратном уравнении с параметром? 

Если перед x2 стоит коэффициент, обязательно проверить, чтобы он не был равен 0. В противном случае уравнение из квадратного превращается в линейное, а это уже совершенно другой алгоритм решений уравнений. 

1. Поскольку по условию должно быть 2 решения, мы получаем, что a \(\neq\) -0,5. 

2. Найдем дискриминант уравнения. Он должен быть строго больше 0, чтобы у уравнения было два решения. 

D = a2 — 4 * (2a + 1) * (3a + 1) = a2 — 24a2 — 20a -4 = -23a2 — 20a — 4

3. Составим неравенство и решим его:

-23a2 — 20a — 4 > 0
23a2 + 20a + 4 < 0
23a2 + 20a + 4 = 0
D = 400 — 4 * 23 * 4 = 400 — 368 = 32
\(a_1 = \frac{-20 + 4 \sqrt{2}}{46} = \frac{2\sqrt{2} — 10}{23}\)
\(a_2 = \frac{-20 — 4\sqrt{2}}{46} = \frac{-2\sqrt{2} — 10}{23}\)

4. 2 + 20a + 4 = 23(a — \frac{2\sqrt{2} — 10}{23})(a — \frac{-2\sqrt{2} — 10}{23})\)

5. Получаем неравенство:

\(23(a — \frac{2\sqrt{2} — 10}{23})(a — \frac{-2\sqrt{2} — 10}{23} < 0\)

6.Тогда  \(a \in (\frac{-2\sqrt{2} — 10}{23}; \frac{2\sqrt{2} — 10}{23})\). Вспомним, что a \(\neq\) -0,5, следовательно, мы получаем ответ \(a \in (\frac{-2\sqrt{2} — 10}{23}; -0,5) \cup (-0,5; \frac{2\sqrt{2} — 10}{23})\).

Ответ: \(a \in (\frac{-2\sqrt{2} — 10}{23}; -0,5) \cup (-0,5; \frac{2\sqrt{2} — 10}{23})\)

Теорема Виета 

Дискриминант — не единственный способ решить квадратное уравнение. Обратимся к теореме Виета. Если нам дано уравнение ax2 + bx + c = 0, то его корни можно найти с помощью следующей системы: 

Теорему Виета удобно использовать, если на корни уравнения наложены дополнительные ограничения. 

Пример 3. При каких значениях параметра a корни уравнения x2 — 3ax — a(a — 1) = 0 удовлетворяют условию x1 = 5x2. 2 — 4a = 0 \rightarrow a(9a — 4) = 0 \rightarrow a = 0, a = \frac{4}{9}\)

5. Мы нашли значения параметра, при которых выполняется условие. Осталось проверить, чтобы при этих значениях у уравнения было два корня. 

a = 0 не подходит, поскольку ограничение \(a \in (-\infty; 0) \cup (\frac{4}{13}; +\infty)\) не включает точку 0. 

\(a = \frac{4}{9}\) подходит, поскольку \(\frac{4}{9} > \frac{4}{13}\). 

Ответ: \(a = \frac{4}{9}\)

Условия на корни квадратного трехчлена 

Однако могут встретиться еще более сложные задания с параметрами. Рассмотрим каждый из этих случаев. 

1. Корни квадратного трехчлена меньше, чем число N. 

Построим параболу. Вспомним, что ветви параболы могут быть направлены или вверх, или вниз. 

Если ветви параболы направлены вверх. Отметим на оси х точку N так, чтобы она лежала правее обоих корней уравнения. Так мы зададим условие, что корни уравнения меньше, чем число N.  

Представим, что мы идем по холмистой местности, и у нас есть ее карта. Имея перед собой плоскую картинку, мы понимаем, как относительно друг друга располагаются точки в пространстве. Но посмотрев на рельеф сбоку, заметим, что точки имеют разную высоту. 

Пусть в точках, где парабола пересекает ось х, будут привалы на экскурсионном маршруте, а в точке N будет смотровая площадка. 

Что можно сказать про смотровую площадку на этой карте? Она находится выше, чем привалы, и лежит правее, чем самая низкая точка рельефа. 

Рассмотрим эти условия на графике. В точке N значение функции f(x) больше, чем в корнях уравнения. Более того, она лежит правее, чем вершина параболы, то есть ее абсцисса больше абсциссы параболы. 

Почему эти условия так важны? Пусть точка N будет лежать левее вершины параболы. Тогда не выполняется условие, что корни меньше, чем N. 

В этом случае на нашем экскурсионном маршруте смотровая площадка будет лежать до привалов.  

А если значение функции в точке N будет меньше, чем в корнях уравнения? Точка N будет лежать между ними. 

В этом случае смотровая площадка окажется между привалами. 

Аналогичным способом можно проследить изменение условий при любом положении точки N на графике. 

Для того чтобы оба корня квадратного трехчлена ax2 + bx + c были меньше, чем число N, необходимо и достаточно выполнение следующих условий: 

Что произойдет, если ветви параболы будут направлены вниз? Наш экскурсионный маршрут немного поменяется: появится гора, а не овраг. 

Где теперь располагается смотровая площадка? Она будет ниже, чем привалы, и дальше, чем самая высокая точка горы. 

Мы можем сделать вывод, что точка N на графике будет лежать правее вершины параболы, а значение функции в ней будет меньше, чем значение функции в корнях уравнения. 

Для того чтобы оба корня квадратного трехчлена ax2 + bx + c были меньше, чем число N, необходимо и достаточно выполнение следующих условий: 

2. Корни квадратного трехчлена больше, чем число N. 

Рассуждаем так же, как и в предыдущей функции, однако теперь точка N перемещается левее параболы. 

Если ветви параболы направлены вверх, то функция в точке N принимает большее значение, чем в корнях уравнения, а сама точка N будет лежать левее параболы. 

Для того чтобы оба корня квадратного трехчлена ax2 + bx + c были больше, чем число N, необходимо и достаточно выполнение следующих условий: 

Теперь направим ветви параболы вниз. Значение функции в точке N будет меньше, чем в корнях уравнения. 

Для того чтобы оба корня квадратного трехчлена ax2 + bx + c были больше, чем число N, необходимо и достаточно выполнение следующих условий: 

С помощью анализа расположения точек на графике функций можно задать условия для любой ситуации, даже если точек будет несколько. 

Алгоритм: как задать любые условия для корней квадратных уравнений с помощью графика? 

Достаточно начертить примерный график функции и расставить на оси х нужные точки. Чтобы составить систему, необходимо: 

1. Определить, куда направлены ветви параболы и задать условие для коэффициента перед x2.
2. Определить, сколько корней имеет уравнение и задать условие для дискриминанта.
3. Определить расположение вершины параболы относительно точек на графике и задать условие для их абсцисс.
4. Определить, какое значение принимает функция в данных точках относительно корней уравнения. 

В итоге должна получиться система, с помощью которой можно решить задачу.

Фактчек
  • Параметр — это буква a, вместо которой можно подставить число. Решить уравнение с параметром — это найти такие числовые значения параметра, при которых условие выполняется. 
  • При решении линейного уравнения ax=b в зависимости от значения коэффициентов может получиться несколько вариантов решений. Если a = 0, b = 0 — бесконечное множество решений. Если a = 0, b \(\neq\) 0 — решений нет. Если a \(\neq\) 0, b \(\neq\) 0 — решением будет \(x = \frac{b}{a}\). 
  • При решении квадратного уравнения обязательно проверять коэффициент перед x2. Если коэффициент будет равен 0, то уравнение станет линейным.
  • При решении квадратного уравнения важно учитывать значение дискриминанта: если он строго больше 0, то корней у уравнения два, если дискриминант равен 0, то у уравнения один корень, если дискриминант меньше 0, то у уравнения нет корней. 
  • Решить квадратное уравнение можно и с помощью теоремы Виета
  • Если в задаче даны дополнительные условия на корни уравнения (например, они должны быть больше или меньше определенного числа), то задать их можно с помощью системы. Неравенства в системе можно составить с помощью анализа примерного графика функций. 

Проверь себя

Задание 1. 
Что такое параметр?

  1. Это буква a, вместо которой можно подставить число.
  2. Это коэффициент перед x2 в квадратном уравнении.
  3. Это переменная х.
  4. Это значение функции в определенной точке. 

Задание 2. 
Дано уравнение ax = b. Сколько решений оно имеет, если a = 0 и b = 0?

  1. Решений нет.
  2. Одно решение.
  3. Бесконечное множество решений.
  4. Невозможно определить количество решений. 

Задание 3. 
При каких значениях дискриминанта уравнение будет иметь корни?

  1. D > 0
  2. D = 0
  3. D < 0
  4. D \(\neq\) 0

Задание 4. 
Корни квадратного уравнения меньше числа А. Где будет лежать вершина параболы относительно точки А?

  1. Справа.
  2. Слева.
  3. Совпадать с точкой А.
  4. Невозможно определить расположение вершины. 

Задание 5. 
Меньший корень квадратного уравнения больше числа А, но меньше числа В. Ветви параболы направлены вниз. Чему будет равно значение функции в точке В?

  1. Значение функции в точке В будет меньше 0.
  2. Значение функции в точке В будет равно 0.
  3. Значение функции в точке В будет больше 0.
  4. Невозможно определить значение функции. 

Ответы: 1. — 1 2. — 3 3. — 4 4. — 2 5. — 3.

Способы решения квадратных уравнений | Статья в журнале «Юный ученый»



Наш проект посвящен способам решения квадратных уравнений. Цель проекта: научиться решать квадратные уравнения способами, не входящими в школьную программу. Задача: найти все возможные способы решения квадратных уравнений и научиться их использовать самим и познакомить одноклассников с этими способами.

Что же такое «квадратные уравнения»?

Квадратное уравнение — уравнение вида ax2+ bx + c = 0, где a, b, c — некоторые числа (a ≠ 0), x — неизвестное.

Числа a, b,c называются коэффициентами квадратного уравнения.

  • a называется первым коэффициентом;
  • b называется вторым коэффициентом;
  • c — свободным членом.

А кто же первый «изобрёл» квадратные уравнения?

Некоторые алгебраические приемы решения линейных и квадратных уравнений были известны еще 4000 лет назад в Древнем Вавилоне. Найденные древние вавилонские глиняные таблички, датированные где-то между 1800 и 1600 годами до н.э., являются самыми ранними свидетельствами об изучении квадратных уравнений. На этих же табличках изложены методы решения некоторых типов квадратных уравнений.

Необходимость решать уравнения не только первой, но и второй степени еще в древности была вызвана потребностью решать задачи, связанные с нахождением площадей земельных участков и с земляными работами военного характера, а также с развитием астрономии и самой математики.

Правило решения этих уравнений, изложенное в вавилонских текстах, совпадает по существу с современным, однако неизвестно, каким образом дошли вавилоняне до этого правила. Почти все найденные до сих пор клинописные тексты приводят только задачи с решениями, изложенными в виде рецептов, без указаний относительно того, каким образом они были найдены. Несмотря на высокий уровень развития алгебры в Вавилоне, в клинописных текстах отсутствуют понятие отрицательного числа и общие методы решения квадратных уравнений.

Вавилонские математики примерно с IV века до н.э. использовали метод дополнения квадрата для решения уравнений с положительными корнями. Около 300 года до н.э. Эвклид придумал более общий геометрический метод решения. Первым математиком, который нашел решения уравнения с отрицательными корнями в виде алгебраической формулы, был индийский ученый Брахмагупта (Индия, VII столетие нашей эры).

Брахмагупта изложил общее правило решения квадратных уравнений, приведенных к единой канонической форме:

ax2 + bх = с, а>0

В этом уравнении коэффициенты, могут быть и отрицательными. Правило Брахмагупты по существу совпадает с нашим.

В Индии были распространены публичные соревнования в решении трудных задач. В одной из старинных индийских книг говорится по поводу таких соревнований следующее: «Как солнце блеском своим затмевает звезды, так ученый человек затмит славу в народных собраниях, предлагая и решая алгебраические задачи». Задачи часто облекались в стихотворную форму.

В алгебраическом трактате Аль-Хорезми дается классификация линейных и квадратных уравнений. Автор насчитывает 6 видов уравнений, выражая их следующим образом:

1) «Квадраты равны корням», т. е. ах2 = bх.

2) «Квадраты равны числу», т. е. ах2 = с.

3) «Корни равны числу», т. е. ах2 = с.

4) «Квадраты и числа равны корням», т. е. ах2 + с = bх.

5) «Квадраты и корни равны числу», т. е. ах2 + bх =с.

6) «Корни и числа равны квадратам», т. е. bх + с == ах2.

Для Аль-Хорезми, избегавшего употребления отрицательных чисел, члены каждого из этих уравнений слагаемые, а не вычитаемые. При этом заведомо не берутся во внимание уравнения, у которых нет положительных решений. Автор излагает способы решения указанных уравнений, пользуясь приемами ал-джабр и ал-мукабала. Его решение, конечно, не совпадает полностью с нашим. Уже не говоря о том, что оно чисто риторическое, следует отметить, например, что при решении неполного квадратного уравнения первого вида Аль-Хорезми, как и все математики до XVII в., не учитывает нулевого решения, вероятно, потому, что в конкретных практических задачах оно не имеет значения. При решении полных квадратных уравнений Аль-Хорезми на частных числовых примерах излагает правила решения, а затем их геометрические доказательства.

Формы решения квадратных уравнений по образцу Аль-Хорезми в Европе были впервые изложены в «Книге абака», написанной в 1202г. итальянским математиком Леонардом Фибоначчи. Автор разработал самостоятельно некоторые новые алгебраические примеры решения задач и первый в Европе подошел к введению отрицательных чисел.

Эта книга способствовала распространению алгебраических знаний не только в Италии, но и в Германии, Франции и других странах Европы. Многие задачи из этой книги переходили почти во все европейские учебники XIV-XVII вв. Общее правило решения квадратных уравнений, приведенных к единому каноническому виду x2 + bх = с при всевозможных комбинациях знаков и коэффициентов b, c, было сформулировано в Европе в 1544 г. М. Штифелем.

Вывод формулы решения квадратного уравнения в общем виде имеется у Виета, однако Виет признавал только положительные корни. Итальянские математики Тарталья,Кардано, Бомбелли среди первых в XVI в. учитывают, помимо положительных, и отрицательные корни. Лишь в XVII в. благодаря трудам Жирара, Декарта, Ньютона и других ученых способ решения квадратных уравнений принимает современный вид.

Рассмотрим несколько способов решения квадратных уравнений.

Стандартные способы решения квадратных уравнений из школьной программы:

  1. Разложение левой части уравнения на множители.
  2. Метод выделения полного квадрата.
  3. Решение квадратных уравнений по формуле.
  4. Графическое решение квадратного уравнения.
  5. Решение уравнений с использованием теоремы Виета.

Остановимся подробнее на решение приведенных и не приведенных квадратных уравнений по теореме Виета.

Напомним, что для решения приведенных квадратных уравнений достаточно найти два числа такие, произведение которых равно свободному члену, а сумма — второму коэффициенту с противоположным знаком.

Пример.x2-5x+6=0

Нужно найти числа, произведение которых равно 6, а сумма 5. Такими числами будут 3 и 2.

Ответ: x1=2, x2=3.

Но можно использовать этот способ и для уравнений с первым коэффициентом не равным единице.

Пример.3x2+2x-5=0

Берём первый коэффициент и умножаем его на свободный член: x2+2x-15=0

Корнями этого уравнения будут числа, произведение которых равно — 15, а сумма равна — 2. Эти числа — 5 и 3. Чтобы найти корни исходного уравнения, полученные корни делим на первый коэффициент.

Ответ: x1=-5/3, x2=1

6. Решение уравнений способом «переброски».

Рассмотрим квадратное уравнение ах2 + bх + с = 0, где а≠0.

Умножая обе его части на а, получаем уравнение а2х2 + аbх + ас = 0.

Пусть ах = у, откуда х = у/а; тогда приходим к уравнению у2 + by + ас = 0, равносильному данному. Его корни у1 и у2 найдем с помощью теоремы Виета.

Окончательно получаем х1 = у1/а и х2 = у2/а.

При этом способе коэффициент a умножается на свободный член, как бы «перебрасывается» к нему, поэтому его называют способом «переброски». Этот способ применяют, когда можно легко найти корни уравнения, используя теорему Виета и, что самое важное, когда дискриминант есть точный квадрат.

Пример.2 — 11х + 15 = 0.

«Перебросим» коэффициент 2 к свободному члену и сделав замену получим уравнение у2 — 11у + 30 = 0.

Согласно обратной теореме Виета

у1 = 5, х1 = 5/2, х1=2,5 ;у2 = 6, x2 = 6/2, x2 = 3.

Ответ: х1=2,5; х2= 3.

7. Свойства коэффициентов квадратного уравнения.

Пусть дано квадратное уравнение ах2 + bх + с = 0, а ≠ 0.

1. Если a+ b + с = 0 (т.е. сумма коэффициентов уравнения равна нулю), то х1 = 1.

2. Если а — b + с = 0, или b = а + с, то х1 = — 1.

Пример.345х2 — 137х — 208 = 0.

Так как а + b + с = 0 (345 — 137 — 208 = 0), то х1 = 1, х2 = -208/345.

Ответ: х1=1; х2 = -208/345 .

Пример.132х2 + 247х + 115 = 0

Т.к. a-b+с = 0 (132 — 247 +115=0), то х1= — 1, х2= — 115/132

Ответ: х1= — 1; х2=- 115/132

Существуют и другие свойства коэффициентов квадратного уравнения. но ихиспользование более сложное.

8. Решение квадратных уравнений с помощью номограммы.

Рис 1. Номограмма

Это старый и в настоящее время забытый способ решения квадратных уравнений, помещенный на с.83 сборника: Брадис В.М. Четырехзначные математические таблицы. — М., Просвещение, 1990.

Таблица XXII. Номограмма для решения уравнения z2 + pz + q = 0. Эта номограмма позволяет, не решая квадратного уравнения, по его коэффициентам определить корни уравнения.

Криволинейная шкала номограммы построена по формулам (рис. 1):

ОВ =AB =

Полагая ОС = р, ED = q, ОЕ = а (все в см), из рис.1 подобия треугольников САН и CDF получим пропорцию

откуда после подстановок и упрощений вытекает уравнение z2 + pz + q = 0, причем буква z означает метку любой точки криволинейной шкалы.

Рис. 2 Решение квадратных уравнения с помощью номограммы

Примеры.

1) Для уравнения z2 — 9z + 8 = 0 номограмма дает корни z1 = 8,0 и z2 = 1,0

Ответ:8,0; 1,0.

2) Решим с помощью номограммы уравнение

2z2 — 9z + 2 = 0.

Разделим коэффициенты этого уравнения на 2, получим уравнение z2 — 4,5z + 1 = 0.

Номограмма дает корни z1 = 4 и z2 = 0,5.

Ответ: 4; 0,5.

9. Геометрический способ решения квадратных уравнений.

Пример.х2 + 10х = 39.

В оригинале эта задача формулируется следующим образом: «Квадрат и десять корней равны 39».

Рассмотрим квадрат со стороной х, на его сторонах строятся прямоугольники так, что другая сторона каждого из них равна 2,5, следовательно, площадь каждого равна 2,5x. Полученную фигуру дополняют затем до нового квадрата АВСD, достраивая в углах четыре равных квадрата, сторона каждого из них 2,5, а площадь 6,25

Рис. 3 Графический способ решения уравнения х2 + 10х = 39

Площадь S квадрата ABCD можно представить как сумму площадей: первоначального квадрата х2, четырех прямоугольников (4∙2,5x = 10х) и четырех пристроенных квадратов (6,25∙ 4 = 25) , т.е. S = х2 + 10х = 25. Заменяя х2 + 10х числом 39, получим что S = 39+ 25 = 64, откуда следует, что сторона квадрата АВСD, т.е. отрезок АВ = 8. Для искомой стороны х первоначального квадрата получим

10. Решение уравнений с использованием теоремы Безу.

Теорема Безу. Остаток от деления многочлена P(x) на двучлен x — α равен P(α) (т.е. значению P(x) при x = α).

Если число α является корнем многочлена P(x), то этот многочлен делится на x -α без остатка.

Пример.х²-4х+3=0

Р(x)= х²-4х+3, α: ±1,±3, α =1, 1-4+3=0. Разделим Р(x) на (х-1):(х²-4х+3)/(х-1)=х-3

х²-4х+3=(х-1)(х-3), (х-1)(х-3)=0

х-1=0; х=1, или х-3=0, х=3; Ответ: х1=2, х2=3.

Вывод: Умение быстро и рационально решать квадратные уравнения просто необходимо для решения более сложных уравнений, например, дробно-рациональных уравнений, уравнений высших степеней, биквадратных уравнений, а в старшей школе тригонометрических, показательных и логарифмических уравнений. Изучив все найденные способы решения квадратных уравнений, мы можем посоветовать одноклассникам, кроме стандартных способов, решение способом переброски (6) и решение уравнений по свойству коэффициентов (7), так как они являются более доступными для понимания.

Литература:

  1. Брадис В.М. Четырехзначные математические таблицы. — М., Просвещение, 1990.
  2. Алгебра 8 класс: учебник для 8 кл. общеобразоват. учреждений Макарычев Ю. Н., Миндюк Н. Г., Нешков К. И., Суворова С. Б. под ред. С. А. Теляковского 15-е изд., дораб. — М.: Просвещение, 2015
  3. https://ru.wikipedia.org/wiki/%D0%9A%D0%B2%D0%B0%D0%B4%D1%80%D0%B0%D1%82%D0%BD%D0%BE%D0%B5_%D1%83%D1%80%D0%B0%D0%B2%D0%BD%D0%B5%D0%BD%D0%B8%D0%B5
  4. Глейзер Г.И. История математики в школе. Пособие для учителей. / Под ред. В.Н. Молодшего. — М.: Просвещение, 1964.

Основные термины (генерируются автоматически): уравнение, квадратное уравнение, свободный член, решение уравнений, корень, число, способ решения, квадрат, коэффициент, решение.

Уравнения с двумя переменными. Как решить квадратное уравнение

Предлагаем вам удобный бесплатный онлайн калькулятор для решения квадратных уравнений. Вы сможете быстро получить и разобраться, как они решаются, на понятных примерах.
Чтобы произвести решение квадратного уравнения онлайн , вначале приведите уравнение к общему виду:
ax 2 + bx + c = 0
Заполните соответственно поля формы:

Как решить квадратное уравнение: Виды корней:
1. Привести квадратное уравнение к общему виду:
Общий вид Аx 2 +Bx+C=0
Пример: 3х — 2х 2 +1=-1 Приводим к -2х 2 +3х+2=0

2. Находим дискриминант D.
D=B 2 -4*A*C .
Для нашего примера D= 9-(4*(-2)*2)=9+16=25.

3. Находим корни уравнения.
x1=(-В+D 1/2)/2А.
Для нашего случая x1=(-3+5)/(-4)=-0,5
x2=(-В-D 1/2)/2А.
Для нашего примера x2=(-3-5)/(-4)=2
Если В — четное число, то дискриманант и корни удобнее считать по формулам:
D=К 2 -ac
x1=(-K+D 1/2)/А
x2=(-K-D 1/2)/А,
Где K=B/2

1. Действительные корни. Причем. x1 не равно x2
Ситуация возникает, когда D>0 и A не равно 0.

2. Действительные корни совпадают. x1 равно x2
Ситуация возникает, когда D=0. Однако при этом, ни А, ни В, ни С не должны быть равны 0.

3. Два комплексных корня. x1=d+ei, x2=d-ei, где i=-(1) 1/2
Ситуация возникает, когда D
4. Уравнение имеет одно решение.
A=0, B и C нулю не равны. Уравнение становиться линейным.

5. Уравнение имеет бесчисленное множество решений.
A=0, B=0, C=0.

6. Уравнение решений не имеет.
A=0, B=0, C не равно 0.

Для закрепления алгоритма, вот еще несколько показательных примеров решений квадратных уравнений .

Пример 1. Решение обычного квадратного уравнения с разными действительными корнями.
x 2 + 3x -10 = 0
В этом уравнении
А=1, B = 3, С=-10
D=B 2 -4*A*C = 9-4*1*(-10) = 9+40 = 49
квадратный корень будем обозначать, как число 1/2 !
x1=(-В+D 1/2)/2А = (-3+7)/2 = 2
x2=(-В-D 1/2)/2А = (-3-7)/2 = -5

Для проверки подставим:
(x-2)*(x+5) = x2 -2x +5x – 10 = x2 + 3x -10

Пример 2. Решение квадратного уравнения с совпадением действительных корней.
х 2 – 8x + 16 = 0
А=1, B = -8, С=16
D = k 2 – AC = 16 – 16 = 0
X = -k/A = 4

Подставим
(x-4)*(x-4) = (x-4)2 = X 2 – 8x + 16

Пример 3. Решение квадратного уравнения с комплексными корнями.
13х 2 – 4x + 1 = 0
А=1, B = -4, С=9
D = b 2 – 4AC = 16 – 4*13*1 = 16 — 52 = -36
Дискриминант отрицательный – корни комплексные.

X1=(-В+D 1/2)/2А = (4+6i)/(2*13) = 2/13+3i/13
x2=(-В-D 1/2)/2А = (4-6i)/(2*13) = 2/13-3i/13
, где I – это квадратный корень из -1

Вот собственно все возможные случаи решения квадратных уравнений.
Надеемся, что наш онлайн калькулятор окажется весьма полезным для вас.
Если материал был полезен, вы можете

В этой статье мы будем учиться решать биквадратные уравнения.

Итак, уравнения какого вида называются биквадратными?
Все уравнения вида ах 4 + bx 2 + c = 0 , гдеа ≠ 0 , являющиеся квадратными относительно х 2 , и называются биквадратными уравнениями. Как видите, эта запись очень похожа на запись квадратного уравнения, поэтому и решать биквадратные уравнения будем используя формулы, которые мы применяли при решении квадратного уравнения.

Только нам необходимо будет ввести новую переменную, то есть обозначим х 2 другой переменной, например, у или t (или же любой другой буквой латинского алфавита).

Например, решим уравнение х 4 + 4х 2 ‒ 5 = 0.

Обозначим х 2 через у (х 2 = у ) и получим уравнение у 2 + 4у – 5 = 0.
Как видите, такие уравнения вы уже умеете решать.

Решаем полученное уравнение:

D = 4 2 – 4 (‒ 5) = 16 + 20 = 36, √D = √36 = 6.

у 1 = (‒ 4 – 6)/2= ‒ 10 /2 = ‒ 5,

у 2 = (‒ 4 + 6)/2= 2 /2 = 1.

Вернемся к нашей переменной х.

Получили, что х 2 = ‒ 5 и х 2 = 1.

Замечаем, что первое уравнение решений не имеет, а второе дает два решения: х 1 = 1 и х 2 = ‒1. Будьте внимательны, не потеряйте отрицательный корень (чаще всего получают ответ х = 1, а это не правильно).

Ответ: — 1 и 1.

Для лучшего усвоения темы разберем несколько примеров.

Пример 1. Решите уравнение 2х 4 ‒ 5 х 2 + 3 = 0.

Пусть х 2 = у, тогда 2у 2 ‒ 5у + 3 =0.

D = (‒ 5) 2 – 4· 2 · 3 = 25 ‒ 24 = 1, √D = √1 = 1.

у 1 = (5 – 1)/(2· 2) = 4 /4 =1, у 2 = (5 + 1)/(2· 2) = 6 /4 =1,5.

Тогда х 2 = 1 и х 2 = 1,5.

Получаем х 1 = ‒1, х 2 = 1, х 3 = ‒ √1,5 , х 4 = √1,5.

Ответ: ‒1; 1; ‒ √1,5; √1,5.

Пример 2. Решите уравнение 2х 4 + 5 х 2 + 2 = 0.

2у 2 + 5у + 2 =0.

D = 5 2 – 4 · 2 · 2 = 25 ‒ 16 = 9, √D = √9 = 3.

у 1 = (‒ 5 – 3)/(2 · 2) = ‒ 8 /4 = ‒2, у 2 = (‒5 + 3)/(2 · 2) = ‒ 2 /4 = ‒ 0,5.

Тогда х 2 = ‒ 2 и х 2 = ‒ 0,5. Обратите внимание, ни одно из этих уравнений не имеет решения.

Ответ: решений нет.

Неполные биквадратные уравнения — это когда b = 0 (ах 4 + c = 0) или же c = 0

(ах 4 + bx 2 = 0) решают как и неполные квадратные уравнения.


Пример 3. Решить уравнение х 4 ‒ 25х 2 = 0

Разложим на множители, вынесем х 2 за скобки и тогда х 2 (х 2 ‒ 25) = 0.

Получим х 2 = 0 или х 2 ‒ 25 = 0, х 2 = 25.

Тогда имеем корни 0; 5 и – 5.

Ответ: 0; 5; – 5.

Пример 4. Решить уравнение 5х 4 ‒ 45 = 0 .

х 2 = ‒ √9 (решений не имеет)

х 2 = √9, х 1 = ‒ 3, х 2 = 3.

Как видите, умея решать квадратные уравнения, вы сможете справиться и с биквадратными.

Если же у вас остались вопросы, записывайтесь на мои уроки. Репетиор Валентина Галиневская.

сайт, при полном или частичном копировании материала ссылка на первоисточник обязательна.

Представление об уравнениях с двумя переменными впервые формируется в курсе математики за 7 класс. Рассматриваются конкретные задачи, процесс решения которых приводит к такому виду уравнений.

При этом они изучаются довольно поверхностно. В программе главный акцент делается на системах уравнений с двумя неизвестными.

Это стало причиной того, что задачи, в которых на коэффициенты уравнения накладываются определенные ограничения, практически не рассматриваются. Недостаточно внимания уделено методам решения заданий типа «Решить уравнение в натуральных или целых числах». Известно, что материалы ЕГЭ и билеты вступительных экзаменов часто содержат такие упражнения.

Какие именно уравнения определяются как уравнения с двумя переменными?

ху = 8, 7х + 3у = 13 или х 2 + у = 7 – примеры уравнений с двумя переменными.

Рассмотрим уравнение х – 4у = 16. Если х = 4, а у = -3, оно будет правильным равенством. Значит, эта пара значений – решение данного уравнения.

Решение любого уравнения с двумя переменными – множество пар чисел (х; у), которые удовлетворяют это уравнение (превращают его в верное равенство).

Часто уравнение преобразовывают так, чтобы из него можно было получить систему для нахождения неизвестных.

Примеры

Решить уравнение: ху – 4 = 4х – у.

В данном примере можно воспользоваться методом разложения на множители. Для этого нужно сгруппировать слагаемые и вынести общий множитель за скобки:

ху – 4 = 4х – у;

ху – 4 – 4х + у = 0;

(ху + у) – (4х + 4) = 0;

у(х + 1) – 4(х + 1) = 0;

(х + 1)(у — 4) = 0.

Ответ: Все пары (х; 4), где х – любое рациональное число и (-1; у), где у – любое рациональное число.

Решить уравнение: 4х 2 + у 2 + 2 = 2(2х — у).

Первый шаг – группирование.

4х 2 + у 2 + 2 = 4х – 2у;

4х 2 + у 2 + 1 — 4х + 2у + 1 = 0;

(4х 2 – 4х +1) + (у 2 + 2у + 1) = 0.

Применив формулу квадрата разности, получим:

(2х — 1) 2 + (у + 1) 2 = 0.

При суммировании двух неотрицательных выражений ноль получится только в том случае, если 2х – 1 = 0 и у + 1 = 0. Отсюда следует: х = ½ и у = -1.

Ответ: (1/2; -1).

Решить уравнение (х 2 – 6х + 10)(у 2 + 10у + 29) = 4.

Рационально применить оценочный метод, выделив полные квадраты в скобках.

((х — 3) 2 + 1)((у + 5) 2 + 4) = 4.

При этом (х — 3) 2 + 1 ≥ 1, а (у + 5) 2 + 4 ≥ 4. Тогда левая часть уравнения всегда не меньше 4. Равенство возможно в случае

(х — 3) 2 + 1 = 1 и (у + 5) 2 + 4 = 4. Следовательно, х = 3, у = -5.

Ответ: (3; -5).

Решить уравнение в целых числах: х 2 + 10у 2 = 15х + 3.

Можно записать это уравнение в таком виде:

х 2 = -10у 2 + 15х + 3. Если правую часть равенства делить на 5, то 3 – остаток. Из этого следует, что х 2 не делится на 5. Известно, что квадрат числа, которое не делится на 5, должен дать в остатке или 1, или 4. Значит, уравнение корней не имеет.

Ответ: Решений нет.

Не стоит расстраиваться из-за трудностей в поиске верного решения для уравнения с двумя переменными. Упорство и практика обязательно принесут свои плоды.

Цели:

  1. Систематизировать и обобщить знания и умения по теме: Решения уравнений третьей и четвертой степени.
  2. Углубить знания, выполнив ряд заданий, часть из которых не знакома или по своему типу, или способу решения.
  3. Формирование интереса к математике через изучение новых глав математики, воспитание графической культуры через построение графиков уравнений.

Тип урока : комбинированный.

Оборудование: графопроектор.

Наглядность: таблица «Теорема Виета».

Ход урока

1. Устный счет

а) Чему равен остаток от деления многочлена р n (х) = а n х n + а n-1 х n-1 + … + а 1 х 1 + a 0 на двучлен х-а?

б) Сколько корней может иметь кубическое уравнение?

в) С помощью чего мы решаем уравнение третьей и четвертой степени?

г) Если b четное число в квадратном уравнение, то чему равен Д и х 1 ;х 2

2. Самостоятельная работа (в группах)

Составить уравнение, если известны корни (ответы к заданиям закодированы) Используется «Теорема Виета»

1 группа

Корни: х 1 = 1; х 2 = -2; х 3 = -3; х 4 = 6

Составить уравнение:

B=1 -2-3+6=2; b=-2

с=-2-3+6+6-12-18= -23; с= -23

d=6-12+36-18=12; d= -12

е=1(-2)(-3)6=36

х 4 — 2 х 3 — 23х 2 — 12 х + 36 = 0 (это уравнение решает потом 2 группа на доске)

Решение . Целые корни ищем среди делителей числа 36.

р = ±1;±2;±3;±4;±6…

р 4 (1)=1-2-23-12+36=0 Число 1 удовлетворяет уравнению, следовательно, =1 корень уравнения. По схеме Горнера

р 3 (x) = х 3 -х 2 -24x -36

р 3 (-2) = -8 -4 +48 -36=0, х 2 =-2

р 2 (x) = х 2 -3х -18=0

х 3 =-3, х 4 =6

Ответ: 1;-2;-3;6 сумма корней 2 (П)

2 группа

Корни: х 1 = -1; х 2 = х 3 =2; х 4 =5

Составить уравнение:

B=-1+2+2+5-8; b= -8

с=2(-1)+4+10-2-5+10=15; с=15

D=-4-10+20-10= -4; d=4

е=2(-1)2*5=-20;е=-20

8+15+4х-20=0 (это уравнение решает на доске 3 группа)

р = ±1;±2;±4;±5;±10;±20.

р 4 (1)=1-8+15+4-20=-8

р 4 (-1)=1+8+15-4-20=0

р 3 (x) = х 3 -9х 2 +24x -20

р 3 (2) = 8 -36+48 -20=0

р 2 (x) = х 2 -7х +10=0 х 1 =2; х 2 =5

Ответ: -1;2;2;5 сумма корней 8(Р)

3 группа

Корни: х 1 = -1; х 2 =1; х 3 =-2; х 4 =3

Составить уравнение:

В=-1+1-2+3=1;в=-1

с=-1+2-3-2+3-6=-7;с=-7

D=2+6-3-6=-1; d=1

е=-1*1*(-2)*3=6

х 4 — х 3 — 7х 2 + х + 6 = 0 (это уравнение решает потом на доске 4 группа)

Решение. Целые корни ищем среди делителей числа 6.

р = ±1;±2;±3;±6

р 4 (1)=1-1-7+1+6=0

р 3 (x) = х 3 — 7x -6

р 3 (-1) = -1+7-6=0

р 2 (x) = х 2 -х -6=0; х 1 =-2; х 2 =3

Ответ:-1;1;-2;3 Сумма корней 1(О)

4 группа

Корни: х 1 = -2; х 2 =-2; х 3 =-3; х 4 =-3

Составить уравнение:

B=-2-2-3+3=-4; b=4

с=4+6-6+6-6-9=-5; с=-5

D=-12+12+18+18=36; d=-36

е=-2*(-2)*(-3)*3=-36;е=-36

х 4 + 4х 3 – 5х 2 – 36х -36 = 0 (это уравнение решает потом 5 группа на доске)

Решение. Целые корни ищем среди делителей числа -36

р = ±1;±2;±3…

р(1)= 1 + 4-5-36-36 = -72

р 4 (-2) = 16 -32 -20 + 72 -36 = 0

р 3 (х) = х 3 +2х 2 -9х-18 = 0

р 3 (-2)= -8 + 8 + 18-18 = 0

р 2 (х) = х 2 -9 = 0; x=±3

Ответ: -2; -2; -3; 3 Сумма корней-4 (Ф)

5 группа

Корни: х 1 = -1; х 2 =-2; х 3 =-3; х 4 =-4

Составить уравнение

х 4 + 10х 3 + 35х 2 + 50х + 24 = 0 (это уравнение решает потом 6группа на доске)

Решение . Целые корни ищем среди делителей числа 24.

р = ±1;±2;±3

р 4 (-1) = 1 -10 + 35 -50 + 24 = 0

р 3 (х) = x- 3 + 9х 2 + 26x+ 24 = 0

p 3 (-2) = -8 + 36-52 + 24 = О

р 2 (х) = x 2 + 7x+ 12 = 0

Ответ:-1;-2;-3;-4 сумма-10 (И)

6 группа

Корни: х 1 = 1; х 2 = 1; х 3 = -3; х 4 = 8

Составить уравнение

B=1+1-3+8=7;b=-7

с=1 -3+8-3+8-24= -13

D=-3-24+8-24= -43; d=43

х 4 — 7х 3 — 13х 2 + 43 x — 24 = 0 (это уравнение решает потом 1 группа на доске)

Решение . Целые корни ищем среди делителей числа -24.

р 4 (1)=1-7-13+43-24=0

р 3 (1)=1-6-19+24=0

р 2 (x)= х 2 -5x — 24 = 0

х 3 =-3, х 4 =8

Ответ: 1;1;-3;8 сумма 7 (Л)

3. Решение уравнений с параметром

1. Решить уравнение х 3 + 3х 2 + mх — 15 = 0; если один из корней равен (-1)

Ответ записать в порядке возрастания

R=Р 3 (-1)=-1+3-m-15=0

х 3 + 3х 2 -13х — 15 = 0; -1+3+13-15=0

По условию х 1 = — 1; Д=1+15=16

Р 2 (х) = х 2 +2х-15 = 0

х 2 =-1-4 = -5;

х 3 =-1 + 4 = 3;

Ответ:- 1;-5; 3

В порядке возрастания: -5;-1;3. (Ь Н Ы)

2. Найти все корни многочлена х 3 — 3х 2 + ах — 2а + 6, если остатки от его деления на двучлены х-1 и х +2 равны.

Решение: R=Р 3 (1) = Р 3 (-2)

Р 3 (1) = 1-3 + а- 2а + 6 = 4-а

Р 3 (-2) = -8-12-2а-2а + 6 = -14-4а

x 3 -Зх 2 -6х + 12 + 6 = х 3 -Зх 2 -6х + 18

x 2 (x-3)-6(x-3) = 0

(х-3)(х 2 -6) = 0

3) а=0, х 2 -0*х 2 +0 = 0; х 2 =0; х 4 =0

а=0; х=0; х=1

а>0; х=1; х=а ± √а

2. Составить уравнение

1 группа . Корни: -4; -2; 1; 7;

2 группа . Корни: -3; -2; 1; 2;

3 группа . Корни: -1; 2; 6; 10;

4 группа . Корни: -3; 2; 2; 5;

5 группа . Корни: -5; -2; 2; 4;

6 группа . Корни: -8; -2; 6; 7.

Завершение квадрата — формула, метод, шаги, примеры

Завершение квадрата — это метод, который используется для преобразования квадратного выражения формы ax 2 + bx + c в вершинную форму a(x — h ) 2 + к. Наиболее распространенное применение завершения квадрата — решение квадратного уравнения. Это можно сделать, переформулировав выражение, полученное после завершения квадрата: a(x + m) 2 + n так, чтобы левая часть была полным трехчленом квадрата. Завершение метода квадрата полезно в:

  • Преобразование квадратного выражения в вершинную форму.
  • Анализ того, в какой точке квадратное выражение имеет минимальное/максимальное значение.
  • График квадратичной функции.
  • Решение квадратного уравнения.
  • Вывод квадратной формулы.

Давайте узнаем больше о заполнении формулы квадрата, его методе и процессе поэтапного заполнения квадрата. Мы обсудим его приложения, используя решенные примеры для лучшего понимания.

1. Что завершает квадрат?
2. Завершение метода квадрата
3. Завершение формулы квадрата
4. Завершение примеров формулы квадрата
5. Решение квадратных уравнений с использованием метода возведения в квадрат
6. Завершение квадратных шагов
7. Как подать заявку на выполнение метода квадрата?
8. Часто задаваемые вопросы о завершении квадрата

Что завершает квадрат?

Завершение квадрата — это метод в алгебре, который используется для записи квадратного выражения таким образом, чтобы оно содержало полный квадрат. Простыми словами, можно сказать, что завершение квадрата — это процесс, в котором рассматривается квадратное уравнение оси 2  + bx + c = 0 и измените его, чтобы записать его в виде a(x + p) 2  + q = 0. Этот метод обычно используется для нахождения корней квадратного уравнения.

Завершение метода квадрата

Наиболее распространенным применением метода квадратов является разложение квадратного уравнения на множители и, следовательно, нахождение корней или нулей квадратного многочлена или квадратного уравнения. Мы знаем, что квадратное уравнение вида ax 2 + bx + c = 0 можно решить методом факторизации. Но иногда факторизация квадратного выражения ax 2 + bx + c сложна или НЕвозможна. Давайте посмотрим на следующий пример, чтобы понять этот случай.

Например:

x 2 + 2x + 3 нельзя разложить на множители, так как мы не можем найти два числа, сумма которых равна 2, а произведение равно 3. В таких случаях мы записываем это в виде a(x + m) 2 + n на завершая квадрат . Поскольку у нас есть целое (x + m) в квадрате, мы говорим, что здесь мы «завершили квадрат». Но как нам завершить квадрат? Давайте разберемся с концепцией подробно в следующих разделах.

Завершение формулы квадрата

Завершение формулы квадрата — это прием или метод преобразования квадратного многочлена или уравнения в полный квадрат с некоторой дополнительной константой. Квадратное выражение в переменной x: ax 2 + bx + c, где a, b и c — любые действительные числа, кроме a ≠ 0, можно преобразовать в полный квадрат с некоторой дополнительной константой, используя формулу квадрата или технику .

Примечание: Заполнение формулы квадрата используется для получения формулы квадрата.

Завершение формулы квадрата — это метод или метод, который также можно использовать для нахождения корней данных квадратных уравнений, ax 2 + bx + c = 0, где a, b и c — любые действительные числа, но a ≠ 0.

Формула заполнения квадрата:

Формула заполнения квадрата: ax 2 + bx + c ⇒ a(x + m) 2 + n

, где m — любое действительное число, а n — постоянный член.

Вместо того, чтобы использовать сложный пошаговый метод для завершения квадрата, мы можем использовать следующую простую формулу для завершения квадрата. Чтобы завершить квадрат в выражении ax 2 + bx + c, сначала найдите:

m = b/2a и n = c — (b 2 /4a)

Подставьте эти значения в: ax 2 + bx + c = a(x + m) 2 + n. Эти формулы выводятся геометрически. Давайте изучим это подробно, используя иллюстрации в следующих разделах.

Завершение примеров формулы квадрата

Вот несколько примеров применения завершения формулы квадрата. Давайте пройдемся по ним, чтобы понять процесс заполнения квадрата.

Пример 1: Используя формулу квадрата, найдите число, которое нужно прибавить к х 2 — 7x, чтобы получить трехчлен с полным квадратом.

Решение:

Данное выражение равно x 2 — 7x.

Метод 1:

Сравнение данного выражения с ax 2 + bx + c, a = 1; b = -7

Используя формулу, член, который должен быть добавлен, чтобы сделать данное выражение трехчленом с полным квадратом, равен
(б/2а) 2 = (-7/2(1)) 2 = 49/4.
Таким образом, из обоих методов член, который следует добавить, чтобы сделать данное выражение идеальным квадратным трехчленом, равен 49/4.

Метод 2:

Коэффициент x равен -7. Половина этого числа равна -7/2. В поисках квадрата,
(-7/2) 2 = 49/4

Пример 2: Используйте формулу квадрата для решения: x 2 — 4x — 8 = 0.

Решение:

2 Метод :

Используя формулу, ax 2 + bx + c = a(x + m) 2 + n. Здесь a = 1, b = -4, c = -8
⇒ m = b/2a = (-4)/2(1) = -2
и, n = c — (b 2 /4a) = -8 — (-4) 2 /4(1) = -12
⇒ х 2 — 4х — 8 = (х — 2) 2 — 12.
⇒ (х — 2) 2 = 12
⇒ (х — 2) = ±√12
⇒ х — 2 = ± 2√3
⇒ x = 2 ± 2√3

Метод 2:

Перенесем постоянный член в другую часть уравнения: x 2 — 4x = 8. Возьмем половину коэффициента x-члена, что равно -4, включая знак, который дает -2. Возведите в квадрат -2, чтобы получить +4, и добавьте это квадратное значение к обеим частям уравнения:

x 2 — 4x + 4 = 8 + 4
⇒ x 2 — 4x + 4 = 12

Этот процесс создает квадратное выражение, которое представляет собой полный квадрат в левой части уравнения. Просто мы можем заменить квадратное на квадратно-биномиальную форму: (x — 2) 2 = 12
Теперь, когда мы завершили выражение для создания бинома идеального квадрата, давайте решим:

(x — 2) 2 = 12
⇒ (х — 2) = ±√12
⇒ х — 2 = ± 2√3
⇒ x = 2 ± 2√3

Ответ: Используя метод квадратов, x = 2 ± 2√3.

Решение квадратных уравнений с использованием метода возведения в квадрат

Дополним квадрат в выражении ax 2 + bx + c, используя квадрат и прямоугольник в геометрии. На основе метода, изученного ранее, коэффициент x 2 должен быть равен «1», взяв «a» в качестве общего множителя. Получаем: ах 2 + Ьх + с = а[х 2  + (б/а)х + (в/а)]. Теперь рассмотрим первые два члена: x 2 и (b/a)x. Рассмотрим квадрат со стороной «x» (площадь которого равна x 2 ). Рассмотрим также прямоугольник длины (b/a) и ширины (x) (площадь которого равна (b/a)x).

Теперь разделите прямоугольник на две равные части. Длина каждого прямоугольника будет b/2a.

Прикрепите половину этого прямоугольника к правой стороне квадрата, а оставшуюся половину к нижней части квадрата.

Для завершения геометрического квадрата не хватает квадрата со стороной b/2a. Квадрат площади [(b/2a) 2 ] должен быть добавлен к x 2 + (b/a)x, чтобы завершить квадрат. Но мы не можем просто сложить, нам нужно вычесть его, чтобы сохранить значение выражения. Таким образом, чтобы составить квадрат:

x 2 + (b/a) x = x 2 + (b/a)x + (b/2a) 2 — (b/2a) 2

= х 2 + (б/а)х + (б/2а) 2 — b 2 /4a 2

Умножение и деление (b/a)x на 2 дает, x 2 + (2⋅x⋅b/2a) + (b/2a) 2 — b 2 /4a 2

Используя тождество, x 2 + 2xy + y 2 = (x + y) 2
Приведенное выше уравнение можно записать как
. x 2 + (b/a) x = (x + b/2a) 2 — (b 2 /4a 2 )

Подставив это в (1): ax 2 + bx + c = a((x + b/2a) 2 — b 2 /4a 2 + c/a) = a(x + b/2a) 2 — b 2 /4a + c = a(x + b/2a) 2 + (c — b 2 /4a)

Это имеет вид a(x + m) 2 + n, где,
м = б/2а
n = c — (b 2 /4a)

Пример:

Мы заполним квадрат в -4x 2 — 8x — 12, используя эту формулу. Сравнивая это с топором 2 + Ьх + с, а = -4; б = -8; c = -12

Найдите значения «m» и «n», используя:
m = b/2a = -8/2(-4) = 1

n = c — (b 2 /4a) = -12 — (-8) 2 /4(-4) = -8

Подставьте эти значения в: ax 2 + bx + c = a(x + m) 2 + n
Мы получаем: — 4x 2 — 8x — 12 = -4(x + 1) 2 — 8

Мы увидим, что мы придем к тому же ответу, используя пошаговый метод, также в следующем разделе.

Завершение шагов квадрата

Чтобы применить метод заполнения квадрата, мы выполним определенный набор шагов. Ниже приведен процесс пошагового заполнения квадрата:

  • Шаг 1: Запишите квадратное уравнение в виде x 2  + bx + c. (Коэффициент x 2  должен быть равен 1.)
  • Шаг 2: Определите половину коэффициента x.
  • Шаг 3: Возьмем квадрат числа, полученного в шаге 1.
  • Шаг 4: Прибавьте и вычтите квадрат, полученный на шаге 2, к x 2 срок.
  • Шаг 5. Разложите полином на множители и примените алгебраическое тождество x 2 + 2xy + y 2 = (x + y) 2  , чтобы построить квадрат.

Как подать заявку на выполнение метода квадрата?

Теперь, когда мы прошли этапы заполнения квадрата в предыдущем разделе, давайте научимся применять метод заполнения квадрата на примере.

Пример: Заполните квадрат в выражении -4x 2 — 8x — 12.

Решение:

Во-первых, мы должны убедиться, что коэффициент x 2 равен «1». Если коэффициент x 2 НЕ равен 1, мы поместим число снаружи как общий множитель. Получим:

-4х 2 — 8х — 12 = -4(х 2 + 2х + 3)
Теперь коэффициент x 2 равен 1.

  • Шаг 1: Найдите половину коэффициента x. Здесь коэффициент «x» равен 2. Половина 2 равна 1.
  • Шаг 2: Найдите квадрат вышеуказанного числа. 1 2 = 1
  • Шаг 3: Добавьте и вычтите указанное выше число после члена x в выражении, коэффициент которого при x 2 равен 1. Это означает, что -4(x 2 + 2x + 3) = -4(x 2 + 2x + 1 — 1 + 3)
  • Шаг 4: Разложите на множители совершенный квадратный трехчлен, образованный первыми тремя членами, используя тождество x 2 + 2xy + y 2 = (x + y) 2 . В этом случае х 2 + 2x + 1 = (x + 1) 2 . Вышеприведенное выражение из шага 3 принимает вид: -4(x 2 + 2x + 1 — 1 + 3) = -4((x + 1) 2 — 1 + 3)
  • Шаг 5: Упростите последние два числа. Здесь -1 + 3 = 2. Таким образом, приведенное выше выражение имеет вид: -4x 2 — 8x — 12 = -4(x + 1) 2 — 8. Оно имеет вид a(x + m) 2 + н. Таким образом, мы завершили квадрат. Таким образом, -4x 2 — 8x — 12 = -4(x + 1) 2 — 8

Примечание. Для завершения квадрата в выражении ax 2 + bx + c

  • Убедитесь, что коэффициент x 2 равен 1.
  • Прибавьте и вычтите (b/2) 2 после члена ‘x’ и упростите.

Учебный трюк Завершение метода квадрата

Вот несколько советов по завершению метода квадрата.

  • Шаг 1: Запишите форму, которую мы хотим получить после заполнения квадрата: a(x + m) 2 + номер
  • Шаг 2: После расширения получаем, ax 2 + 2amx + am 2 + n
  • Шаг 3: Сравните данное выражение, скажем, ax 2 + bx + c, и найдите m и n как m = b/2a и n = c — (b 2 /4a).

Наводящие вопросы:

  • Решите, заполнив квадрат: x 4 — 18x 2 + 17 = 0. Подсказка: Предположим, что x 2 = t.
  • Напишите следующее уравнение вида (x — h) 2 + (y — k) 2 = r 2 , заполнив квадрат. х 2 + у 2 — 4х — 6у + 8 = 0,

☛ Статьи по теме

  • Калькулятор корней
  • Факторизация квадратных уравнений
  • Сумма и произведение корней
  • Природа корней — Примеры
  • Квадратный корень

Часто задаваемые вопросы о завершении квадрата

Что такое завершение квадратного метода?

Завершение квадрата — это метод в математике, который используется для преобразования квадратного выражения формы ax 2 + bx + c в вершинную форму a(x + m) 2 + n. Чаще всего этот метод используется при решении квадратного уравнения, которое может быть выполнено путем перестановки выражения, полученного после завершения квадрата: a (x + m) 2 + n, так что левая часть представляет собой идеальный квадратный трехчлен.

Как проще всего научиться составлять квадрат?

Самый простой способ научиться выполнять метод квадрата — использовать формулу квадрата, a(x + m) 2 + n = a(x + m) 2 + n. Здесь m и n можно рассчитать с помощью следующих формул: m = b/2a и n = c — (b 2 /4a)

Какая польза от заполнения квадрата?

Заполнение формулы квадрата используется для следующих целей:

  • Преобразование квадратного выражения в вершинную форму.
  • Анализ того, в какой точке квадратное выражение имеет минимальное/максимальное значение.
  • График квадратичной функции.
  • Решение квадратного уравнения.

Что добавить при заполнении квадрата?

Если у нас есть уравнение ax 2 + bx, то нам нужно сложить и вычесть (b/2a) 2 , что дополнит квадрат в выражении. Это приведет к [x + (b/a)] 2 — (b/2a) 2 .

Как заполнить квадрат с двумя переменными?

Чтобы понять метод заполнения квадрата с двумя переменными, давайте посмотрим, как заполнить квадрат в выражении с двумя переменными. Рассмотрим выражение с двумя переменными x 2 + y 2 + 2x + 4y + 7. Чтобы завершить возведение в квадрат, мы берем каждый из коэффициентов x и y, делим их значение пополам, а затем возводим его в квадрат. Коэффициент при x = 2, коэффициент при y = 4. Это означает, что (1/2 × 2) 2 = 1 и (1/2 × 4) 2 = 4.

Складываем и вычитаем это к заданному уравнению. Затем переставьте термины, чтобы заполнить квадраты.

x 2 + y 2 + 2x + 4y + 7 + (1 — 1) + (4 — 4) = (x 2 + 2x + 1) + (y 2 + 4y + 4 ) + 7 — 1 — 4 = (x + 1) 2 + (y + 2) 2 + 2

Когда использовать Заполнение квадрата?

Мы используем метод завершения квадрата, когда хотим преобразовать квадратное выражение формы ax 2 + bx + c в вершинную форму a(x — h) 2 + k.

Что такое завершение формулы квадрата?

Завершение формулы квадрата — это формула, необходимая для преобразования квадратного многочлена или уравнения в полный квадрат с некоторой дополнительной константой. Выражается как,
ax 2 + bx + c ⇒ a(x + m) 2 + n, где m и n — действительные числа.

Какая польза от завершения формулы квадрата?

Завершение формулы квадрата используется, когда мы хотим представить квадратный многочлен или уравнение в виде полного квадрата с некоторой дополнительной константой и, таким образом, использовать для факторизации квадратного многочлена.

Что завершают квадратные ступени?

Ниже приведен процесс пошагового заполнения квадрата:

  • Шаг 1: Запишите квадратное уравнение в виде x 2  + bx + c. (Коэффициент x 2  должен быть равен 1.)
  • Шаг 2: Определите половину коэффициента x.
  • Шаг 3: Возьмем квадрат числа, полученного в шаге 1.
  • Шаг 4. Прибавьте и вычтите квадрат, полученный на шаге 2, из числа x 2  .
  • Шаг 5. Разложите полином на множители и примените алгебраическое тождество x 2 + 2xy + y 2 = (x + y) 2  , чтобы построить квадрат.

9.2: Решение квадратных уравнений с использованием свойства квадратного корня

  1. Последнее обновление
  2. Сохранить как PDF 9{2} &=50 \конец{выровнено}\) Шаг 2 : Используйте свойство квадратного корня. Не забудьте написать символ \(\pm\). \(x=\pm \sqrt{50}\) Шаг 3 : Упростите радикал. Перепишите, чтобы показать два решения. \(\begin{array}{l}{x=\pm \sqrt{25} \cdot \sqrt{2}} \\ {x=\pm 5 \sqrt{2}} \\ {}x=5 \sqrt{2}, \:x=-5\sqrt{2}\end{массив}\) 9{2}-27=0\).

    Ответить

    \(y=3 \sqrt{3}, y=-3 \sqrt{3}\)

    Здесь перечислены шаги, необходимые для использования свойства Square Root Property для решения квадратного уравнения.

    Решение квадратного уравнения с использованием свойства квадратного корня

    1. Выделите квадратный член и сделайте его коэффициент равным единице.
    2. Использовать свойство квадратного корня.
    3. Упростите радикал.
    4. 9{2}=36\) Использование свойства квадратного корня. \(z=\pm \sqrt{36}\) Упростите радикальное. \(z=\pm 6\) Перепишите, чтобы показать два решения. \(z=6, \quad z=-6\)

      Проверьте решения:

      Рисунок 9.1.1   Таблица 9{2}=-72\) Использование свойства квадратного корня. \(x=\pm \sqrt{-72}\) Упростите, используя комплексные числа. \(x=\pm \sqrt{72} i\) Упростите радикальное. \(x=\pm 6 \sqrt{2} i\) Перепишите, чтобы показать два решения \(x=6 \sqrt{2} i, x=-6 \sqrt{2} i\) 9{2}+5=17\) Выделить квадратичный член. Умножьте на \(\frac{3}{2}\), чтобы получить коэффициент \(1\). Упрощение. Использование свойства квадратного корня. Упростите радикальное. 9{2}-8=41\).

      Решение :

      9{2}=12\) 9{2}=\frac{5}{9}\)

      Использовать свойство квадратного корня.

      \(x-\frac{1}{3}=\pm \sqrt{\frac{5}{9}}\)

      Перепишите радикал в виде доли квадратных корней.

      \(x-\frac{1}{3}=\pm \frac{\sqrt{5}}{\sqrt{9}}\)

      Упростите радикал.

      \(x-\frac{1}{3}=\pm \frac{\sqrt{5}}{3}\)

      Найдите \(x\).

      \(x=\frac{1}{3} \pm \frac{\sqrt{5}}{3}\)

      Перепишите, чтобы показать два решения.

      \(x=\frac{1}{3}+\frac{\sqrt{5}}{3}, x=\frac{1}{3}-\frac{\sqrt{5}}{3 }\) 9{2}=-12\)

      Использовать свойство квадратного корня.

      \(2 x-3=\pm \sqrt{-12}\)

      Упростите радикал.

      \(2 x-3=\pm 2 \sqrt{3} i\)

      Прибавьте \(3\) к обеим сторонам.

      \(2 x=3 \pm 2 \sqrt{3} i\)

      Разделите обе части на \(2\).

      \(x=\frac{3 \pm 2 \sqrt{3 i}}{2}\)

      Переписать в стандартной форме.

      \(x=\frac{3}{2} \pm \frac{2 \sqrt{3} i}{2}\)

      Упростить.

      \(x=\frac{3}{2} \pm \sqrt{3} i\)

      9{2}=16\)
       
      Выделить квадратичный член.
      Разделите на \(2\), чтобы получить коэффициент \(1\).
      Упрощение.
      Использование свойства квадратного корня.
      Перепишите радикал как часть квадратных корней.
      Рационализируйте знаменатель.
      Упрощение.
      Перепишите, чтобы показать два решения.

      Чек:

      Мы оставляем вам чек.

      Используйте свойство квадратного корня для бинома. \(y-7=\pm \sqrt{12}\)
      Упростите радикальное. \(y-7=\pm 2 \sqrt{3}\)
      Найдите \(y\). \(y=7 \pm 2 \sqrt{3}\)
      Перепишите, чтобы показать два решения. \(y=7+2 \sqrt{3}\)
      \(y=7-2 \sqrt{3}\)
      Использование свойства квадратного корня. \(2 n+1=\pm \sqrt{16}\)
      Упростите радикальное. \(2 n+1=\pm 4\)
      Найдите \(n\). \(2 n=-1 \pm 4\)
      Разделите каждую сторону на \(2\). \(\begin{aligned} \frac{2 n}{2} &=\frac{-1 \pm 4}{2} \\ n &=\frac{-1 \pm 4}{2} \end {выровнено}\)
      Перепишите, чтобы показать два решения. \(n=\frac{-1+4}{2}, n=\frac{-1-4}{2}\)
      Упростите каждое уравнение. \(n=\frac{3}{2}, \quad n=-\frac{5}{2}\)

      Чек:

      Рисунок 9.1.22
       
      Таблица 9.1.7
      Упражнение \(\PageIndex{19}\)

      Решите: \(9 м^{2}-12 м+4=25\). 9{2}+40 н+25=4\).

      Ответить

      \(n=-\frac{3}{4}, \quad n=-\frac{7}{4}\)

      Получите доступ к этому онлайн-ресурсу, чтобы получить дополнительные инструкции и попрактиковаться в использовании свойства Square Root для решения квадратных уравнений.

      • Решение квадратных уравнений: свойство квадратного корня
      • Использование свойства квадратного корня для решения квадратных уравнений

      Ключевые понятия 9{2}=k\), затем \(x=\sqrt{k}\) или \(x=-\sqrt{k}\)или \(x=\pm \sqrt{k}\)

      Как решить квадратное уравнение, используя свойство квадратного корня.
      1. Выделить квадратичный член и сделать его коэффициент равным единице.
      2. Использовать свойство квадратного корня.
      3. Упростите радикал.
      4. Проверьте решения.

      Эта страница под названием 9. 2: Решение квадратных уравнений с использованием свойства квадратного корня распространяется под лицензией CC BY 4.0 и была создана, изменена и/или курирована OpenStax с помощью исходного контента, который был отредактирован в соответствии со стилем и стандартами платформы LibreTexts. ; подробная история редактирования доступна по запросу.

      1. Наверх
        • Была ли эта статья полезной?
        1. Тип изделия
          Раздел или страница
          Автор
          ОпенСтакс
          Лицензия
          СС BY
          Версия лицензии
          4,0
          Программа OER или Publisher
          ОпенСтакс
          Показать страницу TOC
          нет
        2. Теги
          1. источник@https://openstax. org/details/books/intermediate-алгебра-2e

        Ваше пошаговое руководство — Mashup Math

        Что такое «Завершение формулы квадрата» и как ее можно использовать для решения задач?

        Добро пожаловать в это бесплатное руководство, которое прилагается к этому объяснению завершения квадрата! видеоурок, где вы узнаете ответы на следующие ключевые вопросы и информацию:

        • Что такое завершение формулы квадрата?

        • Как решить, заполнив квадрат?

        • Как научиться решать квадратные уравнения, составив квадрат?

        • Что такое завершение квадратных шагов?

        Это полное руководство по заполнению квадрата включает несколько примеров, пошаговое руководство, анимированный видео-мини-урок, а также бесплатный рабочий лист и ключ к ответу.

        *Это руководство к уроку сопровождает нашу анимацию «Объяснение завершения квадрата»! YouTube.

        Хотите больше бесплатных уроков математики и видео? Подпишитесь на наш канал бесплатно!

        Решение путем заполнения квадрата используется для решения квадратных уравнений в следующем виде:

        Обратите внимание, что квадратное число можно преобразовать, вычитая константу c с обеих сторон следующим образом:

        Рисунок 1

        Это два разных способа выражения квадратного числа.

        Имейте это в виду при решении следующих задач:

        Следующий метод меньше похож на формулу и больше похож на

        , выполняя квадратные шаги:

        Пример: Решите следующий квадрат, заполнив квадрат.

        Обратите внимание, что a=1 и b=-6

        Обратите внимание, что a=1 и b=-6, а как насчет константы c?

        Завершение квадрата Шаг 1 из 3: переставьте, если возможно

        Чтобы построить квадрат, вам нужно, чтобы все константы (числа, не привязанные к переменным) были справа от знака равенства.

        В этом примере вы можете получить это, вычитая 9 с обеих сторон и упрощая следующим образом:

        Теперь, когда вы переставили квадратную функцию так, что все константы находятся справа от знака равенства, вы готовы к шагу 2!

        92 равно 9, продолжайте и добавьте 9 к обеим сторонам знака равенства следующим образом:

        Вы можете упростить правую часть знака равенства, добавив 16 и 9.

        Обратите внимание, что вы можете упростить правую часть знака равенства, добавив 16 и 9, чтобы получить 25.

        Теперь вы готовы к последнему шагу!

        Завершение квадрата Шаг 3 из 3: Фактор и решение 92 вместо (x-3)(x-3) очень важно, поскольку позволяет решить задачу следующим образом:

        Упростите, взяв квадратный корень из обеих частей.

        Это то, что осталось после извлечения квадратного корня из обеих частей.

        После извлечения квадратного корня из обеих частей у вас останется x-3 = +/- 5.

        Затем, чтобы получить x отдельно, прибавьте 3 к обеим сторонам следующим образом.

        И чтобы найти решения, просто выполните x = 3 + 5 И x = 3 — 5 , чтобы получить ответ следующим образом:

        Ответ: x= 8 и x = -2

        Этот метод будет применяться к решению любого квадратного уравнения! Давайте быстро рассмотрим выполнение шагов метода квадратной формулы ниже, а затем рассмотрим еще несколько примеров.

        Решение путем выполнения обзора метода квадратных шагов:

        Пример 1:

        Найдите x, заполнив квадрат.

        ШАГ 1/3: ИЗМЕНИТЕ ПРИ НЕОБХОДИМОСТИ

        Начните с перемещения всех констант справа от знака равенства следующим образом:

        92

        Квадратный корень из 8 примерно равен 2,83

        Вот решения!

        Ответ: x=1,83 и x=-3,83

        Пример 2:

        Найдите x, заполнив квадрат.

        В этом последнем примере следуйте 3-шаговому методу квадратной формулы для поиска решений* следующим образом:

        *Обратите внимание, что эта задача будет иметь мнимые решения.

        Шаг 1/3: Переместите константы в правую сторону.

        92 в обе стороны.

        Шаг 3/3: Фактор и решение

        Вы можете получить более подробное пошаговое объяснение того, как решить приведенный выше пример, посмотрев видеоурок ниже, начиная с минуты 7:36.

        Все еще запутались? Посмотрите анимационный видеоурок ниже:

        Посмотрите видеоурок ниже , чтобы узнать больше о заполнении квадрата и увидеть больше пошаговых решений квадратных задач:

        Бесплатный рабочий лист!

        Вы ищете дополнительную практику? Перейдите по ссылкам ниже, чтобы загрузить бесплатные рабочие листы и ключи ответов:

        Заполнение рабочего листа Square Practice:

        НАЖМИТЕ ЗДЕСЬ, ЧТОБЫ ЗАГРУЗИТЬ БЕСПЛАТНЫЙ ТАБЛИЦ

        Есть мысли? Поделитесь своими мыслями в разделе комментариев ниже!

        (Никогда не пропустите блог Mashup Math — щелкните здесь, чтобы получать наш еженедельный информационный бюллетень!)

        Автор: Энтони Персико. Вы часто можете увидеть, как я с радостью разрабатываю анимированные уроки математики, которыми я делюсь на моем канале YouTube  . Или проводить слишком много времени в тренажерном зале или играть на своем телефоне.

        1 Комментарий

        Завершение квадрата — формула, как решить уравнение, примеры шагов

        Как решать уравнения, используя формулу заполнения квадрата с примерами

        Что завершает квадрат?

        Содержание

        • Что завершает квадрат?
        • Как выполнить квадратные шаги
        • Завершение формулы квадрата
        • Решить, заполнив квадрат Примеры
          • Пример
        • Решение квадратных уравнений методом квадрата
          • Пример
        • Заполнение рабочего листа квадрата
        • Почему важно использовать метод квадрата?

        Определение: Завершение квадрата  – это метод, используемый в алгебре для решения квадратного уравнения путем преобразования уравнения стандартной формы ( ax + bx + c ) в уравнение вершинной формы ( a(x-h) + k ), заменив левую часть на совершенный квадратный трехчлен.

        В вершинной форме намного проще решать или находить нули квадратных уравнений, чем в стандартной форме уравнения. Это также помогает найти вершину (h, k), которая будет максимальной или минимальной в уравнении.


        Как выполнить квадратные шаги

        Вот , завершающие квадратные шаги и операции для решения квадратного уравнения в алгебре.

        1. Начните с разложения на
        2. Переместите член c в другую часть уравнения.
        3. Используйте член b , чтобы найти новый член c , который образует идеальный квадрат. Это делается путем деления члена b на 2, возведения в квадрат частного и добавления к обеим частям уравнения.
        4. Найдите свой h , член b, разделенный на два, для идеального квадрата.
        5. Обнулить уравнение.

        Завершение формулы квадрата

        Завершение формулы квадрата вычисляется путем преобразования левой части квадратного уравнения в трехчлен с полным квадратом.

        Например, если мяч брошен и движется по пути завершения квадратного уравнения x + 6x – 8 = 0.

        Максимальная высота мяча или когда мяч находится на земле, будут ответами которое можно найти, когда уравнение имеет вершинную форму. Когда вы закончите квадрат, вы получите уравнение (x+3) – 17 = 0.


        Решите, заполнив квадрат Примеры

        Пример

        Первый пример будет выполнен с приведенным выше уравнением так как он имеет коэффициент 1, поэтому a = 1,

        Давайте решим x + 6x – 8 = 0.

        Шаг № 1 – Переместите член c в другую часть уравнения, используя сложение.

        Шаг № 2 – Используйте член b, чтобы найти новый член c, который образует идеальный квадрат. Это делается путем деления члена b на 2 и возведения в квадрат частного.

        Это число добавляется к обеим частям уравнения для поддержания баланса уравнения. То, что вы делаете с одной стороной, вы делаете с другой стороной.

        Шаг № 3 – Упростить

        Ваш новый совершенный квадрат, h , представляет собой член b, деленный на два. Это связано с тем, что вы разделяете этот термин на две части.

        x 2 + 3x + 3x + 9 = (x+3) 2

        Шаг 4 0


        Решение квадратных уравнений путем заполнения квадрата

        Пример

        Теперь давайте решим квадратное равенство, дополнив квадрат, когда и не равны нулю.

        Давайте начнем со следующего уравнения: 2x + 20x + 8 = 0

        Шаг № 1 – Начните с вынесения члена a, разделите каждый член на 2

        0 Шаг № 2 – 0 Переместите член c в другую часть уравнения, используя вычитание.

        *Вы не вычитаете 4, а вычитаете, поскольку член a был вынесен за скобки. Посмотрите на исходное уравнение, член c равен 8,9.0011

        Шаг №3 – Найдите новый термин c .

        При добавлении нового члена c к обеим сторонам, имейте в виду, что еще раз вам нужно умножить его на факторизованный член a .

        2(x + 10x + 25) = -8 + 50

        Упростить

        2(x + 10x + 25 ) = 42

        4 – Ваш идеальный квадрат h , это член b, разделенный на два. Это связано с тем, что вы разделяете этот термин на две части.

        x 2 + 5x + 5x + 9 = (x + 5) 2

        Шаг #5 — установите уравнение на 0 с помощью вычитания


        Завершение квадратного листа

        Иногда его полезно при изучении новой концепции или набора операций для использования рабочего листа. Вот , заполняющий квадратный рабочий лист , который вы можете загрузить и попрактиковаться в решении набора квадратных уравнений.

        Ответы на эти задачи находятся в этой рабочей таблице, которую вы можете загрузить и проверить свою работу.


        Почему важно использовать метод квадрата?

        Используя , завершая метод квадратов , квадратное уравнение находится в вершинной форме. В таком виде легче найти нули уравнения.

        Эта информация часто является начальной или конечной точкой для многих уравнений. Например, когда объект касается земли.

        Также легко найти вершину уравнения. Вершина является либо точкой максимума, либо точкой минимума уравнения. Это отлично подходит, когда вы хотите найти максимальную прибыль или объем при определенных ограничениях.

        Решение квадратичных уравнений, заполнив квадрат

        Факторингротсформляграфайнгсамплс

        Purplemath

        Квадратное уравнение в последнем примере предыдущей страницы было:

        ( x — 2) 2 — 12 = 0

        . -сторону этого уравнения можно умножить и упростить до:

        x 2 − 4 x − 8

        Но мы все равно не смогли бы решить уравнение, даже в квадратном формате таким образом, потому что он не учитывается и не готов к извлечению квадратного корня.

        Содержание продолжается ниже

        MathHelp.com

        Завершение квадрата

        Единственная причина, по которой мы смогли решить эту задачу на предыдущей странице, заключалась в том, что они уже поместили все элементы размером x в квадрат, поэтому мы могли переместить строго числовую часть уравнения с другой стороны знака «равно», а затем квадратный корень из обеих сторон. Они не всегда будут форматировать вещи так красиво, как это. Итак, как нам перейти от обычного квадратного уравнения, такого как приведенное выше, к уравнению, которое готово для извлечения квадратного корня?

        Нам нужно «заполнить квадрат».


        Вот как бы мы решили последнее уравнение на предыдущей странице, если бы они не отформатировали его для нас.

        Как отмечалось выше, это квадратное число не учитывается, поэтому я не могу решить уравнение с помощью факторизации. И они не дали мне уравнение в форме, готовой для извлечения квадратного корня. Но у меня есть способ манипулировать квадратным числом, чтобы привести его в форму, готовую для извлечения квадратного корня, чтобы я мог решить.

        Во-первых, я помещаю свободное число в другую часть уравнения:

        x 2 -4 x -8 = 0

        x 2 -4 x = 8

        , затем я смотрю на коэффициент x -что я смотрю на коэффициент на x -что я смотрю на коэффициент . −4 в этом случае. Я беру половину этого числа (, включая знак ), что дает мне -2. (Мне нужно отслеживать это значение. Это упростит мою работу в дальнейшем.)

        Затем я возвожу это значение в квадрат, чтобы получить +4, и добавляю это квадратичное значение к обеим частям уравнения:

        х 2 − 4 х + 4 = 8 + 4

        х 2 − 4 х + 4 = квадратное выражение, которое создает квадратичный процесс на 12

        9 левая часть уравнения. Я могу разложить на множители или просто заменить квадратное число квадратно-биномиальной формой, которая представляет собой переменную x вместе с половинным числом, которое я получил раньше (и заметил, что мне понадобится позже), которое было −2. В любом случае, я получаю уравнение с квадратным корнем:

        ( x − 2) 2 = 12

        (я знаю, что это «−2» в скобках, потому что половина −4 равна −2. Отметив знак, когда я нахожу половину коэффициент, я помогаю себе не перепутать знак позже, когда буду переводить в квадратно-биномиальную форму.)

        (Кстати, этот процесс называется «дополнением квадрата», потому что мы добавляем член для преобразования квадратное выражение во что-то, что действует как квадрат бинома, то есть мы «завершили» выражение, чтобы создать бином с идеальным квадратом.)

        Теперь я могу извлечь квадратный корень из обеих частей уравнения, упростить и решить:

        ( x − 2) 2 = 12

        Используя этот метод, я получаю тот же ответ, что и раньше. ; а именно:


        Для решения этого уравнения есть один дополнительный шаг, потому что старший коэффициент не равен 1; Сначала мне нужно разделить, чтобы преобразовать старший коэффициент в 1. Вот мой процесс:

        2 x 2 − 5 х + 1 = 0

        х 2 − (5/2) х + 1/2 = 0

        x 2 — (5/2) x = -(1/2)

        Теперь, когда у меня есть все члены с переменными с одной стороны, со строго числовым членом с другой стороны, я готов завершить квадрат с левой стороны. Во-первых, я беру коэффициент линейного члена (вместе со знаком), -(5/2), умножаю на половину и возвожу в квадрат:

        (1/2) × [-(5/2)] = -(5/4)

        (−(5/4)) 2 = 25/16

        Затем я добавляю это новое значение к обеим частям, преобразую в квадратно-биномиальную форму слева и решаю:

        x 2 − (5/2) x + 25/16 = −(1/2) + 25/16

        ( х — 5/4) 2 = 17/16

        кв[( x − 5/4) 2 ] = ± sqrt[17/16]

        x — 5/4 = ± sqrt[17]/4

        x = 5/4 ± sqrt[17]/4

        Два члена в правой части последней строки выше могут быть объединены под общим знаменателем, и это часто («обычно»?) как будет записан ответ, особенно если в инструкции к упражнению есть условие «упростить» окончательный ответ:

        x = (5 ± sqrt[17])/4


        В другом месте у меня есть урок по решению квадратных уравнений путем завершения квадрата. Этот урок (повторно) объясняет шаги и дает (больше) примеров этого процесса. Он также показывает, как квадратичная формула может быть получена из этого процесса. Если вам нужны дополнительные инструкции или практика по этой теме, пожалуйста, прочитайте урок по гиперссылке выше.

        Кстати, если вам не сказали, что у вас есть для заполнения квадрата, вы, вероятно, никогда не будете использовать этот метод на практике при решении квадратных уравнений. Либо какой-то другой метод (например, факторинг) будет более очевидным и быстрым, либо формула квадратов (рассмотренная далее) будет проще в использовании. Однако, если в вашем классе рассматривалось заполнение квадрата, вы должны ожидать, что вам потребуется показать, что вы можете заполнить квадрат, чтобы решить квадратное уравнение на следующем тесте.


        Вы можете использовать виджет Mathway ниже, чтобы попрактиковаться в решении квадратных уравнений, заполнив квадрат. Попробуйте введенное упражнение или введите свое собственное упражнение. Затем нажмите кнопку и выберите «Решить, заполнив квадрат», чтобы сравнить свой ответ с ответом Mathway. (Или пропустите виджет и перейдите на следующую страницу.)

        Пожалуйста, примите «предпочтительные» файлы cookie, чтобы включить этот виджет.

        (Нажмите «Нажмите, чтобы просмотреть шаги», чтобы перейти непосредственно на сайт Mathway для платного обновления.)



        URL: https://www.purplemath.com/modules/solvquad3.htm

        Страница 1Страница 2Страница 4Страница 5Страница 6

        Предварительное вычисление по алгебре — Как решать уравнения с квадратными корнями?

        Спросил

        Изменено 3 года, 3 месяца назад

        Просмотрено 5к раз

        $\begingroup$

        Я борюсь с явно простой задачей:

        Найти $x,y$:

        $I: \; \sqrt x + \sqrt y=8, \quad \quad II: \; \sqrt{xy}=15$

        Я пробовал разные способы (поставить $\sqrt x$ из $I$ в $II$) для решения этих уравнений, но всегда застревал. 2-(a+b)x+ab=0,$ 92-4\cdot1\cdot15}}2=\frac{8\pm2}2=5$ или $3$

        $\endgroup$

        $\begingroup$

        Совет: используйте формулы Виета. $ $ $ $ $ $

        $\endgroup$

        $\begingroup$

        Кажется, я только что понял (иногда уже достаточно задать вопрос публично, чтобы найти ответ…)

        $II: \; \sqrt{xy}=15 \Leftrightarrow\sqrt x=\frac{15}{\sqrt y}$. Поместите в $I$:

        $\frac{15}{\sqrt y}+\sqrt y=8 \Leftrightarrow15+y=8 \sqrt y \Leftrightarrow y-8 \sqrt y+15=0$ 92 — 4\times15} = \pm 2\\ \подразумевает \begin{случаи} \sqrt{x} = \frac12 \left((\sqrt{x} + \sqrt{y}) + (\sqrt{x} — \sqrt{y})\right) = \frac12 ( 8 \pm 2 ) & = 4 \pm 1.\\ \sqrt{y} = (\sqrt{x} + \sqrt{y}) — \sqrt{x} = 8 — \sqrt{x} & = 4\mp 1. \end{case}$$

        $\endgroup$

        $\begingroup$

        Нет необходимости в формулах Виета или каких-либо ухищрениях, так как работает метод исключения.

    Добавить комментарий

    Ваш адрес email не будет опубликован. Обязательные поля помечены *